Clinical Dissertation

  • Uploaded by: Azam Arzoo
  • 0
  • 0
  • April 2020
  • PDF

This document was uploaded by user and they confirmed that they have the permission to share it. If you are author or own the copyright of this book, please report to us by using this DMCA report form. Report DMCA


Overview

Download & View Clinical Dissertation as PDF for free.

More details

  • Words: 22,862
  • Pages: 118
Clinical based case reports By

Dr Azam Arzoo M.B.B.S (Bangladesh) Clinical Dissertation of Nephrology Sheffield Kidney Institute University of Sheffield

Under Supervision of Professor A M El Nahas Professor of Nephrology, University of Sheffield Sheffield Kidney Institute, Northern General Hospital Sheffield United Kingdom

1

PAGES

CONTENTS List of abbreviations Acknowledgement Dedication Abstract 1 Ward: RUE 1 multiple myeloma associated with hoarseness of voice: case report 2 ARF due to alcoholism: a case report

4 5 6 7 08-15 16-20

3

Reflux nephropathy: a case report

21-25

4

Anemia management in CKD: a case report

26-30

5

ARF secondary to fluid overload: a case report

31-35

2 Ward: Vickers- 3 1 CKD related risk factor for CVD: a case report

. . 36-39

2

IgA nephropathy and management: case report

49-48

3

Acute kidney injury due to insect bite: case report

49-53

4

Diabetic nephropathy and hypertension: case report

54-57

5

P-ANCA positive vasculitis: case report

58-62

3 Ward:

Vickers 2 and Transplantation 1

Assessment and management of comorbid diseases in CKD: case 63-68 report

2

Wegener’s granulomatosis: case report

3

Pulmonary hypertension in patients with ESRD on HD: case 75-78 report

4

Risk of mortality for ESRD patients on dialysis: case report

79-83

5

ADPKD: case report

84-88

69-74

2

6

Ureteric stent( advantages and disadvantages): case report

89-95

7

ATN after Transplantaion report case

96-100

8

Psychotic disorders, after transplant: case report

101-105

9

Delayed graft function and acute tubular necrosis (ATN): case report

106-110

10

Post-transplant IgA nephropathy: case report

111-115

11 References

116-118

3

List of abbreviations AKI: CKD: ESRD: ESRF: DN: HTN: CRF: ANCA: ANA: GFR: ADPKD: GN: RRT: HD: PD: DM: IgAN: BP: IHD: S.Cr: SKI: DBP: SBP: ACEi: ARB: NSAID: DGF: CAPD: ATN: AR: CT: CIT: US: ASA: KUB: CVS: PMH: Hb: MCD: NIDDM: RUE : WG : JVP :

Acute Kidney Injury Chronic Kidney Disease End stage of Renal Disease End Stage of renal failure Diabetic Nephropathy Hypertension Chronic Renal Failure Anti Neutrophil cytoplasmic antibody Anti Nuclear Antibody Glomerular filtration Rate Autosomal Dominant Polycystic Kidney Disease Glomerulonephritis Renal replacement Theraphy Haemodialysis Peritoneal Dialysis Diabetes Mellitus Immunoglobulin A Nephropathy Blood Pressure Ischaemic Heart Disease Serum Creatinine Sheffield Kidney Institute Diastolic Blood Pressure Systolic Blood Pressure Angiotensin Converting Enzyme Inhibitor Angiotensin Receptor Blocker Non Steroidal Anti Inflamatory Drug Delayed Graft Function Continious Ambulatory Peritoneal Dialysis Acute Tubular Necrosis Acute Rejection Computer Tomography Scan Cold Ischemic Time Ultrasonogram American Society of Anesthesiology Kidney Urinary Bladder Cardiovascular System Past Medical History Haemoglobin Minimal Change Disease Non Insulin Dependent Diabetes Mellitus Renal Unit E floor Wegerner’s Granulomatosis Jugular Venous Pressure

4

ACKNOWLEDGEMENT First and foremost, praise be to Almighty ALLAH, the creator of the world, the beneficent and the most merciful. Without his help and guidance, this work, and every other work, would not be possible

My sincerest appreciation to Prof AM EL NAHAS, for affording me the opportunity to pursue this thesis in the department of Nephrology, Sheffield Kidney Institute, University of Sheffield and for his guidance and supervision all through the preparation of this thesis.

I would like to thank Dr Lutfi, Dr Kossi, Dr Brown, Dr Brenann, Dr Kawar, Dr Othman, Dr Parvez, Dr Amino Bello and Dr Ghada Said M. Omar

Finally, my deepest gratitude is due to my parents, brothers and sister for their love, prayer and continues encouragement throughout the course.

5

Dedicated to My mother Late Tasliman Nisa who scarifies her whole life to make me a doctor and dreamt for me to get specialized degree from England And her wish was that “Before being a good doctor I should be a good human being”

6

Abstract Chronic kidney disease (CKD) is a worldwide public health problem with an increasing incidence and prevalence, poor outcomes, and high cost. Outcomes of chronic kidney disease include not only kidney failure but also complications of decreased kidney function and cardiovascular disease. Current evidence suggests that some of these adverse outcomes can be prevented or delayed by early detection and treatment. Unfortunately, chronic kidney disease is underdiagnosed and undertreated, in part as a result of lack of agreement on a definition and classification of its stages of progression Objective: To distinguish the risk factors leading to kidney diseases through investigating different patients with kidney diseases (acute and chronic), and to describe the management of these patients with explaining the natural course of the disease. Design of the study: 20 in-patients diagnosed either with acute or chronic kidney disease, were participated in this study. The patients were randomly selected from Vickers-2, Vickers-3 and RUE in Northern General Hospital, Sheffield Kidney Institute, Sheffield, England, UK. The patients were fully aware of the objectives of the study. A complete physical examination for each patient was completed on the first session. A weekly follow-up and case report recording for each patient was followed for the length of the study (4 weeks). Main outcome measures: Most likely that the patients diagnosed with CKD of any stage converted into ESRD and therefore the patient might need renal replacement therapy in form of peritoneal or haemo-dialysis (PD or HD) or renal replacement therapy. This is only meant that patient might improve his or her quality of life but he/she complication nevertheless. Conclusion: Risk factors such as hypertension, diabetes mellitus, or aging (age >55) were the most common leading factors to chronic kidney diseases, however the risk of end stage renal disease among those patients was very low.

7

A case report: multiple myeloma associated with hoarseness of voice

Introduction: Multiple myeloma belongs to a group of diseases known as the plasma cell dyscrasias, in which an uncontrolled proliferation of plasma cells, which elaborates a specific immunoglobulin molecule. These single clones of cells proliferate diffusely in the bone marrow, with associated bone destruction, but can give rise to a localized mass in bone, soft tissues, or both. Multiple myeloma accounts for 1% of all malignancies, with an incidence of four cases per 100,000, and accounts for 10% of hematologic malignancies. There are two important variants of multiple myeloma: solitary bone plasmacytoma and extramedullary plasmacytoma. Solitary bone plasmacytoma (SBP) is a localized intraosseous lytic lesion without marrow plasmacytosis. Extramedullary plasmacytoma is a plasma-cell tumor manifesting outside the bone marrow without evidence of systemic disease. Extramedullary plasmacytoma and solitary bone plasmacytoma have a better prognosis, and the patients are at least 10 years younger than those with multiple myeloma. Most authors think that SBP, extramedullary plasmacytoma, and multiple myeloma are different manifestations of the same disease. Progression of SBP and extramedullary plasmacytoma to multiple myeloma is the most important feature affecting the prognosis. The disseminated form, multiple myeloma, is by far the most frequent and has the worst prognosis of the three types.

Case Summary: A 59-year-old man admitted to the hospital after a sever deteriorating hoarseness of voice since 15 days. Patient came in RUE unit with complains of haemoptysis, rigth side pleuric chest pain and hoarseness of voice associated with shortness of breath. In addition the patient started coughing up blood since last 2 weeks. The cough is more in the morning with slight blood coming out with sputum. The patient advised by his general practitioner (GP) to analyze sputum sample. Bone marrow biopsy showed plasma cells

8

Case Report: Name of the patient: B B Sex: male Age: 59 DOB: 25/09/1947: Present complain : Aware of diagnosis Feeling better HOPI Patient came in RUE unit with complain of haemoptysis, side pleuric chest pain and hoarseness of voice. Patient started coughing up blood since last 2 weeks. The cough more in morning blood is coming slightly coming with sputum. Patient went to his GP and GP advised him for sputum sample. Also complained right side pluraic chest pain. He also has history of shortness of breath, also noticed hoarseness’ of voice since last 15 days. Weight loss last 3 months but his appetite is good. No joint pain, no epitaxis no rash, no blood in urine, urine volume normal. No recent flu like symptom, no orthopnea. Dietian advised him low phosphorus diet and no added salt diet also fluid restriction. Reqested medic consider prescribing phosphate binder. Note hypercalcemia. Patient is now in dialysis using tunnel line on right atrium. No evidence of pneumothorax. Advising femoral line because tunnel has a complication of low chance of recovery.

9

PMH: MGUS IgA nephropathy lamda in serum and urine 2001 had bone marrow biopsy COPD disease with bullae Emphysema left decoritication CKD 2 Pleural effusion left mini thoracotomy and plural biopsy, no malignancy found Personal history: Life long smoker, stoped in july in 06 Used to smoked 10/15/day Moderate alcohol consumption 6-8 pints/week

Allergy history No allergy history Family history Not significant Drug history Renagel 800mg BD Frusemide 250mg OD Na HCO3 50 mg BD

10

Salbutamol 2.5 mg OD Paracetamol 1gm sos Recent therapy CORTICOSTERIODS CHEMOTHERAPY STEM CELL TRANSPLANTATION THALIDOMIDE BORTEZOMIB LENALIDOMIDE RADIATION THERAPY

On examination: BP- 125 /70 mm of Hg Pulse- 90 R/R-18 br/min Euvolumic JVP No sign of vasculitis No peripheral Sacral odema

CVS Heart sound normal

Respiratory 11

Chest clear Left thoracotomy scar

Abdomen Soft non tender

Investigation: 17/07/07 HB 11.6 HCT 0.38 WCC 5.3 PLTS 176000 ALK.PO4 109 TOTAL PROTEINS 68 NA 139 K 4.9 UREA 37.6 CR 478 CA 2.62 PO4 2.06 Renal function feb 2007 Urea- 6.4 and Crt-108 Worsening renal failure with Biopsy report Plasma cells constitute about 80% of the present and lie in confluent masses, extensive 80% infiltration by myloma

Differential diagnosis: 12

Multiple Myeloma Amyloidosis Other Malignancy

DISCUSSION Multiple myeloma is a plasma cell neoplasia. It is an incurable but treatable disease characterized by replacement of the bone marrow with malignant plasma cells, bone marrow destruction and paraprotein formation. This paraprotein is also known as the Mcomponent. This molecule is a homogeneous, partial or complete immunoglobulin. Based on the type of the M-component, multiple myeloma can be subdivided into the following types: IgG, IgA, IgD, IgE and IgM. Common clinical findings in multiple myeloma are hypercalcemia, anemia, renal damage, impaired production of normal immunoglobulins, osteoporosis and lytic bone lesions. The patient presented in this case had all of these symptoms. Additionally, multiple myeloma patients are at an increased risk of bacterial infections. The most common myeloma types are IgG and IgA, followed by IgD, IgE and IgM. There is also light chain disease where only light chains are synthesized, either kappa or lambda. Another variant is a nonsecretory myeloma with no M-component secretion.

Bone Involvement The bone lesions are formed as the tumor grows in the bone marrow and invades the hard bone creating the characteristic multiple small lytic bone lesions. The patient presented in this case already had back pain and on skeletal survey was found to have multiple lytic lesions. The severity of these lesions ranges from osteopenia to fractures. This bone destruction usually presents as bone pain, frequently in the lumbar spine.

Renal Disease Multiple myeloma causes renal failure in nearly 25% of patients. Hypercalcemia is the most common cause of renal failure, as it leads to tubulointerstitial disease. There is

13

expression of the calcium receptor in the thick ascending limb of the loop of Henle where hypercalcemia inhibits reabsorption of calcium, magnesium and sodium chloride. In addition, hypercalcemia causes hypercalciuria via an increased filtered calcium load and suppression of parathyroid hormone release with a consequent reduction in calcium reabsorption.

Myeloma Cells and Bone Marrow Disease Myeloma cells are of the same type in any given person and produce large quantities of the same immunoglobulin protein, called monoclonal M protein, or paraprotein. When blood or urine is processed in electrophoresis, these M proteins show up as a "spike" in the results. The myeloma cells proliferate in the bone marrow and do not leave space for the normal B lymphocytes to develop and produce adequate immunoglobulins. This makes the patient prone to infections. Other cell lines are produced in smaller quantity. Fewer white blood cells and red blood cells are produced, and therefore patients present with anemia and susceptibility to bacterial infections.

Hyperviscosity Syndrome When the protein concentration in blood becomes very high the blood becomes very viscous, leading to flow problems. This is called hyperviscosity syndrome and some signs and symptoms of it are shortness of breath, confusion, chest pain, spontaneous bleeding and blurred vision. IgA myeloma causes hyperviscosity because of the predisposition of the IgA molecule to form dimers. Increased protein in the blood is what produces the classical roleaux finding (stacking of red blood cells) in the peripheral blood smear. The patient in this case was found to have multiple myeloma of the IgA type and she was experiencing hyperviscosity syndrome with symptoms of bleeding, shortness of breath and confusion. Both staging systems are of importance because they give prognostic values and aid in deciding when to start treatment. Treatment is tailored to each individual depending on several things, including the stage, physical exam and symptoms. The typical treatment is high dose chemotherapy and bone marrow stem cell transplantation. Adjunctive therapy 14

for myeloma is radiation therapy. Bisphosphonates are used against skeletal events because they inhibit bone resorption via action on osteoclasts or osteoclast precursors. Erythropoietin induces erythropoiesis and can be used to treat anemia from myeloma or from the chemotherapy. Plasmaphoresis may be done to treat the symptoms of hyperviscosity syndrome. Hydration sometimes is enough to treat hypercalcemia. Corticosteroids can be used to reduce swelling in patients with spinal cord compression due to multiple myeloma. Patients with acute renal failure benefit from dialysis, plasmaphoresis, management of hypercalcemia and avoidance of nephrotoxic medications. The standard chemotherapy regimen (VAD) combines vincristine, doxorubicin and dexamethasone. Another regimen commonly employed is (VBMCP) vincristine, bischloroethylnitrosourea, melphalan, cyclophosphamide and prednisone. MP (melphanprednisone) has also been used. Treatments currently in research include the use of interferon alfa to prolong remission. Thalidomide has recently been found to be an active agent in the management of multiple myeloma. Bortezomib (Velcade) is a reversible proteosome inhibitor that has also been used with good results. After several trials, the patient had the best response to a regimen involving regular hemodialysis and thalidomide-dexamethasone treatment. 1. Radiation therapy, used by some authors to the chest in combination with systemic therapy resulted in regression in the amount of effusion but no remission of myeloma. 2. Makino et al used intracavitary interferon-a (given at 5 MIU dissolved in 100 ml normal saline alternate days), and the patient’s effusion resolved after six treatments. Unfortunately, the patient died of myeloma about two months later, therefore this cannot be considered as standard. 3. The mainstay of therapy as of now is systemic chemotherapy. Reference: [1]

15

Acute Renal Failure (ARF) due to alcoholism accombined with nonsteroidal antiinflammatory drug (NSAID): a case report Back ground: Case of reversible acute renal failure (ARF) following binge drinking together with the transient use of a nonsteroidal antiinflammatory drug (NSAID) is described. After binge drinking, the patient experienced hyperdipsia, and the volume of his urine decreased. Subsequently, he took an NSAID to relieve systemic joint pain associated with low grade fever, and then he had complete anuria. One day after taking the NSAID, the patient admitted to the hospital after diagnosed with severe renal dysfunction accompanied by severe liver damage (blood urea nitrogen and creatinine concentrations were 57 and 5.4 mg/dl, respectively). The impaired renal function progressed over the first three hospital days, as reflected by an elevated creatinine concentration to ll.6 mg/dl. Nine treatment sessions of hemodialysis were, therefore, required to recover the loss of renal function. The present case suggests that binge drinking may be a potential risk factor for ARF.

Case summary: A case of middle aged male who developed swelling and weakness of muscles in the lower limbs following a heavy binge of alcohol is being reported. He had myoglobinuria and developed acute renal failure for which he was dialyzed. Acute alcoholic myopathy is not a well recognized condition and should be considered in any intoxicated patient who presents with muscle tenderness and weakness.

Case report: Name of the patient: S R G Age: DOB: 05/07/68

Present complain:

16

Recent depression He has on haemodialysis Feeling well and no pain

HOPI: Patient came in hospital with history of aneuric and diagnosed ARF due to Rhabdomyolysis with Crt 598, Urea- 31.6, K-4.5, CK-804040 Early patient admitted in psychiatry dept in Barnsley Hopspital then he transferred in renal unit due to deterioration of renal function. Patient came back from Thailand had history of food poisoning over there after he so depressed and taking fluoxetine. In Thiland for 3 month early this year and he had history of unprotective sexual contact with female sex worker. No sign of compartment syndrome.

PMH Depression Ex alcoholic 5-6 pints/day Food poisoning while in Thialand early this year Developed diahorrea last 10 days Personal history Smoking 10-20 cigarate per day Alcohol consume 5-6 pint/day Family history Not significant Drug history: Omeprazole 10mg od

17

Gliclazide 160mg bd Lisinopril 2.5mg od Metformin 1gm bd Sodium bicarbonate Calcichew 1.25 gm tds

On examination BP-160/90 mm of Hg Pulse: 78/min R/R- 17 br/min Peripheral edema present CVS Heart sound normal JVP not raised Respiratory Chest clear Abdomen Soft non tender

Investigation 14/07/07 HB 10.7 NA 138 K 5.4 CA 2.12 PO4 1.08 ALBUMIN 38 AST 18 HCO3 20

18

USS Normal size of kidney

Diagnosis ARF due to Rhabdomyolysis

Discussion: Acute renal failure has been reported to be induced by alcohol ingestion alone. The case in this report showed evidence of non-traumatic rhabdomyolysis due to binge drinking. Severe muscle damage and extremely high levels of serum CK and myoglobin were observed in the cases of rhabdomyolysis-related ARF. In the present patient, the slight elevation of serum CKand myoglobin suggests that muscle damage might be less than that in the above-mentioned cases. Gabow et al reported that ARF is seen in 33% of patients with nontraumatic rhabdomyolysis, and dialysis therapy is required in only 15% of the cases They showed that anion gap (28±14 mEq//) and phosphorus (7.3+3.5 mg/dl) in patients with rhabdomyolysis-related ARFwere higher than those in patients with non-rhabdomyolysisrelated ARF ( 1 7±6 mEq//, and 4.5± 1.5 mg/dl, respectively) In the current report, the anion gap and phosphorus were 18.7 mEq// and 3.4 mg/dl, respectively. These results suggest that the patient had rhabdomyolysis, yet the muscle damage was not severe enough to induce dependent ARF requiring dialysis therapy. This, therefore, suggests that the reported case is not typical rhabdomyolysis related ARF. Experimental and clinical studies have revealed that volume depletion, which leads to activation of the pressor mechanism, appears to be a crucial factor in precipitating rhabdomyolysis induced ARF. Dehydration is a frequent consequence of heavy alcohol ingestion, because it is frequently accompanied by water dieresis, poor food and fluid intake and vomiting, especially in cases of severe binge alcohol ingestion. Furthermore, the majority of cases with rhabdomyolysis-induced ARF have manifested apparent body weight loss. The present patient, however, did not have a poor fluid intake nor suffered

19

from any vomiting or apparent body weight loss before taking the NSAID, but he had hyperdipsia, dark urine and a decreased urine volume after binge drinking, suggesting that he was already in a mild dehydrated state before taking the NSAID. The increases in his hematocrit and total protein levels were slight, and FENa was more than 1%, making it difficult to suggest that the ARF was due to binge drinking alone. In fact, the patient had an episode of complete anuria following NSAID ingestion. The present case strongly suggests a linkage between alcohol-induced rhabdomyolysis and accelerates the impairment of renal hemodynamics. In summary, in the present study, it had been reported that a patient with acute renal failure following binge drinking in which case an episode of complete anuria following NSAID suggests that NSAID ingestion can accelerate the progression of impaired renal function induced by mild rhabdomyolysis, mild dehydration, and severe liver damage resulting in acute renal failure requiring dialysis therapy. It is therefore very important that physicians and pharmacists be alerted to the potential risk of NSAID induced acute renal failure when NSAIDs are ingested following binge drinking, even if patients have no severe muscle damage. In the future, further case reports will surely support our recommendation that the use of NSAIDs should be avoided.

Reference: [2], [3], [4]

20

Reflux nephropathy : a case report Introduction: Reflux nephropathy (RN) as a cause of hypertension and end stage renal disease is being increasingly recognised. It leads to advanced renal failure in 30% of children and in 1520% of adults. The incidence of hypertension in children with RN ranges from 5 to 30%. It is important to realise that early detection of RN and long-term management of reflux can prevent both hypertension and end stage renal disease. Therefore, there is a need for increased awareness of this entity amongst the practising physicians so that cases are picked up early and by proper therapy adverse sequel of hypertension, chronic renal failure and end stage renal disease are prevented.

Case summary: A 63-year-old female suffer from hypertension was brought to Northern General Hospital with complaints of pain in abdomen, anorexia and was feeling unwell last 2 months. She had past medical history of urinary tract infection at the age of 5 years. Now she is feeling lethargy with weight loss. The patient lost her appetite for 3 consecutive days. She also feeling back pain around the lower spine area. She told that unable to walk much at home because of leg weakness. She could not movement facial muscle properly and limb as well. The patient neither was walking or speaking properly due to lethargy. On examination, she had a high blood pressure of 200/120 mmHg

Case report: Sur name : G D R Age: 63, Gender: Female

Present complain: feeling unwell last 2 months Lethargic for 2 month Loss of appetite and does not eat properly last 3 days Back pain around the lower spine 21

Log weakness unable to walk much at home

History of present illness: she is feeling unwell with lethargy and also losing weight day by day. She informed me that she does not ate for 3 days. She also feeling back pain around the lower spine area. She told that unable to walk much at home because of leg weakness. She could not movement facial muscle properly and limb as well She cant speak properly and also have evidence of weight loss.

Past medical history: Reflux nephropathy, HTN

Medication: Calcium gluconate 10% i/v Sodium bicarbonate 200mg tds 15 units actrapid in 50ml 50% dextrose Gelofusin 500ml i/v Augmentin 1.25 gms i/v One alpha .25mcgm od Social history: Lives with husband Not smokes since 27 yrs age She is working as a shop assistant and also she keeps cat/dog

On examination: BP: 127/56, Heart rate: 100 beat per minute

22

CVS: Heart sound normal Respiratory system: Clear both lung Abdomen: soft tender lower abdomen

CNS: 3, 4, 6 intake 5 intakes 7 intakes 8-12 intakes

Investigation: 11/07/07 HB 12.1 HCT .36 WCC 5.3 PLATELETS 166000 ALK. PHOSPHATASE 105 TOTAL PROTEINS 61 NA 128 K 8.7 HCO3 24 UREA 24.2 CREATININE 428 USG: Bilateral small kidney, cortical scarring, no hydronephrosis Urea: 18.2, Creat: 449, K: 5.1

23

ECG: tall T wave, broad QRS D/D: Acute rénal fêlure ---- 1. HTN 2. Reflux nephropathy CKD--- dehydration, sepsis

Discussion: In the recent literature, a lot of controversy surrounds the exact aetiopathogenesis of hypertension in RN. In this connection, the role of intra-renal reflux (IRR) should be emphasised. It has been shown that VUR alone does not lead to renal scarring. Renal scarring occurs only when VUR is associated with IRR. Factors favouring IRR are (i) association of UTI, (ii) association of high pressure VIR, (iii) the anatomical factors like compound and simple papillae. The compound papillae are more susceptible to IRR than simple papillae, due to anatomy of their orifice. How, the renal scarring leads to hypertension are still not thoroughly understood. In this connection the role of plasma renin activity needs to be discussed. In some patients with RN and hypertension the expected age related decrease in PRA, was not observed indicating that high PRA levels are contributing to hypertension. Another important factor is a focal increase in renin secretion and this can be demonstrated by analysing the segmental renal vein PRA. This segmental hyper secretion of renin may not be reflected in the renal vein PRA and yet they may be playing vital role in causing hypertension\. The reflux appears to be primary in our case. Whether this primary reflux is due to congenial abnormalities of the ureter and ureter orifices such as low ratio of intramucosal ureteric length to orifice diameter (Normal ratio 5: 1) could not be determined in our case as the parents were reluctant for cystoscopy which is necessary to identify such a congenital abnormality. For the diagnosis of RN, various imaging techniques have evolved both to demonstrate reflux as well as scarring. IVP can show scarring, but it can be normal in few cases. Dimercaptosuccinic acid (DMSA) scan is more sensitive than IVP in demonstrating scars. Medical Management of hypertension and primary reflux nephropathy consists of following plan: (a) Keeping the urine sterile by continuous prophylactic chemotherapy so as to prevent fresh scarring. In this report the patient was

24

put on prophylactic chemotherapy with nitrofurantoin successfully. The drugs most commonly used are sulphamethoxozole -trimenthoprim, nitrofurantoin and sulphisoxazole. The prophylactic dose is one half of therapeutic dose. How long to continue the prophylactic antibiotic therapy? However, there is another view that if the reflux persists, the age of the patient should be an important determinant in deciding about the discontinuation of long-term chemotherapy. In late childhood and adolescent the disappearance of reflux, especially more than Grade III reflux is very much unlikely and in such a situation, chemo prophylaxis may be discontinued and surgery should be considered as a choice. (b) Hypertension should be treated by drugs, which decrease the renin production as the increased production of renin is shown to be the important factor in pathogenesis of hypertension. Hence captopril is very useful in hypertension. However, other drugs like propronol, methyl dopa have also been found useful. (c) Episodes of intercurrent infection may occur despite prophylactic chemotherapy. This should be vigorously treated with antibiotics, the type of antibiotics being used depending upon the culture and antibiotic sensitivity. The role of surgery in the management of RN is beset with controversy. Surgical success rate for elimination of reflux is quiet high for all grades of VUR: 90-100% for Grades I and II, 93-99% for Grade III and 50-60% for Grade IV and V. The patients showing VUR with or without demonstrable scarring should be given prophylactic chemotherapy to keep urine sterile (The scar may take several months to be demonstrable either by IVP or DNISA scan.

Reference: [5]

25

Anemia management in CKD: a case report Introduction Anemia is common in chronic kidney disease (CKD) due to a state of erythropoietin deficiency. Erythropoietin therapy has been used for approximately 20 years to correct anemia in CKD and to improve both subjective and objective outcomes. Guidelines that establish a hemoglobin (Hb) goal for anemia correction in CKD patients are largely based on observational data. Controversy still exists, however, because outcomes have not been consistent with various degrees of anemia correction. The number of prospective randomized trials investigating the effects of anemia correction on cardiovascular (CV) morbidity and mortality in CKD patients, an already high-risk group, is limited. With respect to improving CV outcomes in the CKD population, the currently available trial data caution against raising Hb levels in CKD patients to approach more "normal" physiologic ranges. The disappointing experience with the trial data must be weighed against the beneficial associations of erythropoietin therapy that have been generated from observational data. Establishing the ideal target Hb ranges for anemia correction in CKD patients remains a dynamic process and leaves many gray areas to be further elucidated. Chronic kidney disease (CKD) has been linked to higher heart failure (HF) risk. Anemia is a common consequence of CKD, and recent evidence suggests that anemia is a risk factor for HF. In this study a case reported to show the association between CKD and anemia.

Case summary: A 82-year-old man with hypertension was initially feeling tired and breathless from last few days. His blood report suggesting severe anaemia 4.6 g/d. renal ultrasound revealed severe bilateral hydronephrosis. In addition, he had history of prostectomy 5 years back and CKD-5. He also has Pedal oedema and clubbing. He was referred to our institution 26

for further evaluation and management of his persistent symptom of severe anaemia, dry cough and mild epitasis for last few months, shortness of breath, loss of appetite and weight loss.

Case report: Surname: L H First name: R Age: 82 yrs Male

Present complain: Feeling tired and breathless from last few days

HOPI: Normally he is well and no medical illness. Look after his wife for last 2 months who had a stroke, feel loss of appetite and weight loss. For last few days feeling tired and shortness of breath.. Today feel more unwell. His blood report suggesting severe anaemia 4.6 g/d. He also complained about dry cough and mild epitasis for last few months. He also denied previous renal failure, urinary symptom and peptic ulcer symptom.

PMH: HTN Prostate operation 5 years back CKD stage 5 Drug history: Venofer 100mg /month Neorecormon 4000i.u 2/week Atorvastatin 40mg od Alfacalcidol 1mcg od

27

Sodium bicarbonate 50mg One alpha 0.25mcg od Ramipril 2.5 mg Allergy history: NAD O/E: Pale looking BP: 106/55, Pulse: 80, Heart rate-100/min, Sats 96% Pedal oedema present Thin comfortable, Clubbing-present Dehydration Lung: Clear CVS: Normal HS Abdomen: Soft, non tender P/R: no melena Catheter: Turbid urine

Investigation: HB 9.1 WCC 10.2 PLTS 182 MCV 87.1 NA 132

28

K 4.6 UREA 10.2 CR 186 Ca 2.0 PO4 2.1 ALK PO4 58 ALBUMIN 38

ECG- No evidence of hyperkalaemia yet Urine dipstick: Blood 2+, Protein- 2+, Leu 2+ USS: Right Kidney measure 10.1 cm and is hydronephrotic renal pelvis measuring 1.5 cm Left kidney measure 8.9 cm and also grossly hydronephrotic and proximal ureter dialated as well. Left renal pelvis measure 3.5 cm. Bladder catheter in situ No mass lesion seen Comment: Bilateral severe hydronephrosis, cortical thinning probably long standing.

Discussion: Anaemic patients with chronic renal failure should receive treatment with recombinant human erythropoietin (r-HuEPO, Eéoetin) to maintain hemoglobin levels over 11 g/dL with an acceptable target of 12 to 12.5 g/dL, according to recommendations from the European practice guideline for management of anemia in patients with chronic renal failure [33] and the National Kidney Foundation K/DOQI clinical practice guidelines for anemia of chronic kidney disease [27]. Benefits of adequate hemoglobin levels had been established in patients undergoing dialysis, and are supposed to be relevant also in CKD patients. In addition, anemic patients should receive iron supplementation in order to maintain serum ferritin levels above 100 μg/L and transferrin saturation above 20%.

29

The CHOIR and CREATE studies, together with previous randomized controlled trials in patients on dialysis, indicate that we should aim for only partial correction of anemia in patients with CKD, and that 115 g/l (11.5 g/dl) is a reasonable upper limit for a hemoglobin target. It is important to recognize that, because of the difficulty of maintaining hemoglobin concentrations within a narrow window, reducing the upper limits of hemoglobin targets is likely to increase the proportion of patients with hemoglobin concentrations less than 110 g/l (11.0 g/dl), and perhaps even the proportion with levels below 100 g/l (10.0 g/dl). The ongoing Trial to Reduce Cardiovascular Events with Aranesp Therapy (TREAT), which allows a hemoglobin level as low as 90 g/l (9.0 g/dl) in one treatment arm, should provide important information about acceptable lower limits.

Conclusion In conclusion, further evidence was found that the concomitant presence of either CKD or anemia increased the risk of dying in the hospital or of being readmitted within 30 days among patients hospitalized with heart failure. The association persisted after controlling for other factors associated with adverse outcomes in these patients.Patients with anemia caused by CKD are at higher risk of death or cardiovascular events when the target hemoglobin level is 135 g/l rather than 113 g/l. Reference: [6], [7]

30

ARF secondary to Fluid overload: a case report Background: Fluid overload is a chronic, troublesome problem in many patients on hemodialysis. These patients suffer from hyperdipsia with inability to excrete water. Angiotensinconverting enzyme inhibitor (ACEI) has been shown to decrease thirst and interdialytic weight gain in 2-4 weeks of usage. It was suggested that long-term ACEI may not continue to suppress inappropriate thirst and fluid intake after 6 months in hemodialysis patients.

Case summary: A 19-year-old male patient with untreated seizure disorder, presenting with Nausea, vomiting, loss of appetite and weight loss for 2 weeks, and impaired renal function – Urea- 213, Creat 743, K-4.2 Usually fit and well started with nausea, vomiting for 2-3 weeks and also has history of haemetemesis. He also complain about haematuria and dysuria, He mentioned that he suffer from decreased urine output. He also was complaining of joint pain but not redness or swelling of joint. He had productive cough with slight haemoptysis 2 weeks ago. No recurrent nose bleeding reported. Patient was referred with lethargy with increased urea and Creatinine. USS abdomen showed right kidney 10.1 cm and left kidney 9.3 cm. No hydronephrosis. Renal biopsies was arranged but cancelled twice because of increase bleeding time in spite of having DDAVP. Urea and Creatinine remain stable throughout his stay in the hospital. Follow up is arranged in the low clearance clinic in 3 weeks time.

Case report: Name: B S M Age: DOB:30/08/1988 31

Present complain: Nausea, vomiting, loss of appetite and weight loss for 2 weeks Impaired renal function – Urea- 213, Creat 743, K-4.2 HOPC: Usually fit and well started with nausea, vomiting for 2-3 weeks and also has history of haemetemesis. He also complain about haematuria and dysuria, He told me that he also has decreased urine output. He has also complaining joint pain but not redness and sweeling of joint. He also informed me has cough with sputum but had slightly haemoptysis 2 weeks ago. Otherwise no recurrent nose bleeding. Patient is being refer with feeling unwell and had increased urea and Creatinine. USS abdomen showed right kidney 10.1 cm and left kidney 9.3 cm. No hydronephrosis. Renal biopsy arrange but cancelled twice because of increase bleeding time inspite of having DDAVP. Urea and Creatinine remain stable through out his stay in the hospital. Follow up is arranged in the low clearance clinic in 3 weeks time.

PMH: He had episode of constant vomiting for 2 months 2 years ago No further investigation and treatment at that time has taken.

Drug history: Frusemide 160mg bd Prednisolone 15mg od One alpha 250mcg Pravastatin 20mg od Septrin 200mg od Inj. Eprex 2000 i.u 3/week Spironolactone 500mg od Oral antacid syrup from GP either nil else Allergy history:

32

Nil Family history: Lives with parents Smokes 10/day Occasionally drinks alcohol. No family history of renal disease and DM 2 cousins have hearing loss and wear hearing aids O/E: BP: 153/86, Sats-98%, Weight 71.4, Pulse-104/min CVS: Systolic murmur present Lung: Clear Abdomen: Soft non tender

Investigation: 16/07/07 HB 9.3 WCC 13.1 PLTS 454000 MCV 96.3 ESR 108 PT 16 APTT 40.2 PTH 123 UREA 20.3 CR 319 CA 2.61 PO4 1.8 K 4.7 NA 136 USS: 33

Both kidney measure appro 10.1 cm Right 9.3 cm Left No abnormality of kidney and no hydronephrosis Bleeding time: more than 10 minutes Urine Dipstick Blood Protein more than 300 mg/dl Glucose negative D/D ARF RPGN

Discussion: Volume overload should be suspected in patients complaining of dyspnea, chest discomfort, orthopnea, paroxysmal nocturnal dyspnea, or progressive decrease in exercise tolerance. It may also be asymptomatic. Physical findings could include jugular venous distention, hepatojugular reflux, pulmonary rales, wheezing (in “cardiac asthma”), and S3 or S4, ascites, and peripheral edema. Patients with chronic kidney failure may also have significant volume overload even in the absence of the above symptoms and signs. Chest films may show evidence of pulmonary edema or may be subtle, showing only prominent pulmonary vasculature. The same findings may occur with heart failure, liver failure and various other conditions, so the patient’s change in weight over time is critical. Over weeks to months, these patients may lose lean body mass due to malnutrition and can develop fluid overload with relatively little change in weight. Therefore, serial assessment of patients’ lean body mass is also critical.

Contributors include:

34



Excess salt intake



Progressive kidney damage (nephrosclerosis)



Fluid retention from blood pressure medications



Inadequate diuretic therapy.

Consider fluid overload for sudden unexplained gains in weight, refractory hypertension, peripheral edema or shortness of breath. These may be secondary to the above causes. Hyponatremia, developing as a result of water retention in excess of sodium retention, may also be a marker for volume overload in the above setting. Management: •

Patients should be weighed at every visit



Dietary sodium restriction to 2 gm/d



Loop diuretics, and if refractory to twice a day dosing, consider adding thiazide-type diuretics



If advanced kidney failure, consider initiation of dialysis.

Reference: [2]

35

Chronic kideny disease (CKD) related risk factor for cardiovascular disease (CVD): a case report

Back ground: Patients with CKD have many of the traditional risk factors for CVD, including older age, hypertension, dyslipidemias, diabetes mellitus, and physical inactivity (Menon, Gui, & Sarnak, 2005). Age is the only one of these traditional risk factors, shared by many patients with CKD, that is not modifiable. Other nonmodifiable CVD risk factors include gender, family history, and hereditary factors such as race. Hypertension, dyslipidemias, diabetes mellitus, smoking, and physical inactivity, risk factors, that adversely affect other risk factors, such as obesity and hypertension, can all be managed or treated, thus are considered modifiable risk factors (Grundy et al., 1999). In addition to traditional CVD risk factors, individuals with CKD must also be assessed for nontraditional risk factors. Many of the complications of CKD, when not adequately treated, may lead not only to more rapid progression of kidney disease but, also, to increased cardiovascular morbidity and mortality (NKF, 2005). These unique CKDrelated risk factors for the development of CVD include anemia, proteinuria/microalbuminuria, disturbances of mineral metabolism, extra cellular volume overload, malnutrition, inflammation, elevated homocysteine levels, and elevated creactive protein levels (Abramson, Jurkoutz, Vaccarino, Leveintraub & McClellan, 2003; Cullerton et al., 1999; Sarnak et al., 2003; Shlipak, Fried, Stehman-Breen, Siscovick, & Newman, 2004). The Kidney Disease Outcome Quality Initiative (K/DOQI) Guidelines recommend consideration of all of these unique risk factors and suggest that efforts to reduce CVD risk should be initiated early in the course of CKD to reduce morbidity and mortality (NKF, 2005).

Case summary: A 75-year-old man of British origin, was admitted to renal unit at Northern General Hospital for further evaluation of chronic kidney disease (CKD). He has no history of

36

diabetes mellitus or hypertension but nocturia 4 times per night. He had history of CKD with progressive incline of creatinine from 136 mmol/l in 2005 to 200 mmol/l in December 2000. Lately a creatine of 153 mmol/l and GFR 38ml/min (CKD) was reported.

Case report: Name: D P Age: 75 Occupation: retired Gender: male HOPI: The patient suffer from shortness of breath, wheeze, hotness, sweaty and dizziness upon standing. Also he felt chest tightness across the chest. There is no history of DM, haematuria but nocturia 4 times per night. Pheripheral oedema is intermittently a problem through is currently well control with just 2 mg of bumetanide. There is no definitive history of orthopnea or paroxymal nocturnal dyspnea. Though he is markedly dyspnea on walking a few yards. H/O CKD with creatinine climbing from 136 mmol/l in 2005 to 200 mmol/l in December. From this clinic visit he had a creatine of 153 mmol/l and GFR 38ml/min (CKD) Past history: CKD, AF, CF No history of DM, stroke, weakness, numbness, coughs, diarrohea, vomiting and leg swollen. Medication: there is recent NSAID used, no allergy drugs. Bisoprolol fumarate 3.75 mg tab Bumetanide 1 mg tab Ramipril 5 mg

37

Bumetanide 1mg Warfarin WBS 1mg Simvastatin 20 mg Omacor 1g Personal history: Ex smoker who drink 5 pints 2 times weekly through was previously a heavier drinker. He lives a lone but has a stair O/E: Pulse BP CVS: S1 and S2 heart sound normal. JVP not visible. Respiratory: Trachea centrally. Few basal crepitation present. GIT: abdomen is soft , no tender and rigidity. Investigation: Na- 139, K- 4, Urea- 10.1, Creat- 147, HCO3- 33, Ca- 2.32, PO4- 1, CaPO4- 2.32, AlkPO4- 87, AST- 40, Bilirubin- 17 Hb- 15.4, WCC- 10.2, MCV-94.3, MCH- 31.1, MCHC- 32.9

Discussion: There are guidelines available to direct the practitioner in reducing a patient's cardiovascular risk factors. The National High Blood Pressure Education Program of the NIH presented its Seventh Report of the Joint National Committee (JNC) on Prevention, Detection, Evaluation, and Treatment of High Blood Pressure in 2004 (USDHHS, 2004). Like its predecessors, these JNC 7 Guidelines provide an evidence-based approach to the prevention and management of hypertension, a major risk factor in the development of CVD as well as CKD (Chobanian et al., 2003). The American Heart Association (AHA) Guidelines for Primary Prevention of

38

Cardiovascular Disease and Stroke: 2002 Update: Consensus Panel Guide to Comprehensive Risk Reduction for Adult Patients without Coronary or Other Atherosclerotic Vascular Diseases provides a framework to assist primary care providers in assessment, management, and follow-up of patients who may be at risk for but who have not yet manifested cardiovascular disease (Pearson et al., 2003). Although the JNC 7 Guidelines and the AHA guidelines provide a framework for comprehensive care for the general population of patients at risk for CVD, they do not specifically address all of the CKD-related nontraditional CVD risk factors. The Third Report of the National Cholesterol Education Program (NCEP) Expert Panel on Detection, Evaluation, and Treatment of High Blood Cholesterol in Adults (ATP III) Executive Summary was released in 2001 (ATP III, 2001). Based on the ATP I and APT II, these were updated, evidenced-based clinical guidelines. They provided direction for cholesterol testing and management that expanded the indications for intensive cholesterol-lowering therapy in clinical practice and applied findings from the most recent clinical trials. While ATP III maintained attention to intensive treatment of patients with CHD, its major new feature was a focus on primary prevention in persons with multiple risk factors. Because of the focus on individuals with multiple CHD risk factors, the guidelines can be applied to the population of patients with CKD. However, they do not specifically address this group or CKD-related risk factors. In 2005, in an effort to more effectively address the risk of CVD in patients with CKD, the National Kidney Foundation (NKF) developed the K/DOQI Guidelines for Cardiovascular Disease in Dialysis Patients (NKF, 2005). The K/DOQI Guidelines recommend that CVD risk factor reduction should be initiated early to reduce morbidity and mortality associated with CKD. The guidelines for treatment and prevention are specific to individuals with Stage 5 CKD, and the extent to which they can be applied to persons in the earlier stages is unclear.

Reference: [8], [9], [10]

39

IgA nephropathy management: case report Back ground: IgAN is the most common type of glomerulonephritis in the world. Between 15 and 40 percent of adults and children diagnosed with IgAN eventually progress to (ESRD). Despite the need for effective treatment strategies, very few Randomized Control Trial for IgAN have been performed. The most effective therapies for IgAN appear to be corticosteroids, ACEi, that contain a high concentration of omega 3 fatty acids. While ACEi are generally well tolerated with minimal side effects, the use of high dose steroids over a long course of therapy is often associated with significant morbidity

Case summary: A 36-year-old man was referred to the Department of Nephrology at the Northern General Hospital (Vickers 3) because of an episode of asymptomatic gross proteinuria suggesting nephritic syndrome with a serum albumin of 21, serum protein of 51.2; clinically she had edema in mild shin on review clinic. The patient’s Alanine Transferase which returned back to normal. In addition her Anti Nuclear Antibody and Ig A antibody were positive. The patient had no family history of renal disease. Biopsy results showed.

Case report: Name: B P Age 60 yrs Chief complain: post biopsy, passing urine HOPI: The patient was feeling unwell recently. She also complained of back pain of dullache nature over the last year and, taking pain killer. She attached anxiety last year and last

40

night she had physiotherapy. After holiday in Turkey in October 2006 she began to complain arthralgia, ankle swelling and severe lethargy. She did have transient upper GI symptom which settle with OPI. She had went a normal OGD and colonoscopy in 2006. Her urine dipstick was positive for protein in a 24 hrs. urine collection at 3.22 for in 24 hrs of protein. Suggesting nephritic syndrome with a serum albumin of 21, serum protein of 51.2, clinically she had odema in mild shin on review clinic. Her ALT which her now settle back to normal. In condition, her ANA was positive, Crt-54, Na- 136, Urea-4.8. She also positive Ig A antibody, low Ca 2.17 She denies any diarrohea, haematuria, previous recurrent UTI, renal stone, flank abdominal pain, she denies facial rashes, red dry eyes, sore throat. She has not had a blood transfusion in past. She denies an any past history of IHD, DM,CVA, malignancy, asthma, COPD Family history: there is no history of kidney disease. She only smoke for a year or when she is very young but has not smoke last 50 yrs. She takes very minimum amount of alcohol. She did biopsy and waiting for biopsy report. HPH: hypothyroidism, proteinuria, microscopic haematuria, HTN Treatment Alfacalcidol 1 mcg od Gliclazide 40mg od Neorecormon 4000i.u / week Clonazepam 50 mcg od Lactulose + senna O/E: Pulse 72 beats/min BP 130/70 mm of Hg CVS: S1 and S2 heart sound normal. JVP not visible. Respiratory: Trachea centrally. Chest clear

41

GIT: abdomen is soft, no tender and rigidity

Investigation: HB 10.8, WCC 12.1, PLTS 196, MCV 91.6, NA 136 K 4.6, UREA 10.2, CR 199, HCO3 21, CA 2.1 PO4 1.45, ALK.PO4 58, ALBUMIN 38

Discussion: Patients with IgAN usually present with hematuria. Approximately 40% to 50% of patients have macroscopic hematuria frequently accompanied by an infection of the upper respiratory tract and, less often, pneumonia, gastroenteritis, or urinary tract infection. The episode is usually brief—about 24 hours—but can last as long as 1 week. This feature usually occurs in children and in patients under 40 years of age, and loin pain often accompanies the hematuria. Microscopic hematuria, usually with proteinuria, constitutes the other common initial presentation in another 30% to 40% of patients. Macroscopic hematuria can complicate the course of about 20% to 25% of patients in this subgroup. This presentation is more common in older patients but is observed in patients of all ages. As many as 20% of patients with IgAN present with severe azotemia that represents long-standing disease. Acute renal failure occurs in less than 10% of patients, and the nephrotic syndrome is uncommon, occurring only in approximately 5% of all patients, usually children and adolescents. The diagnosis of IgAN currently can be made only by kidney biopsy. The spectrum of light microscopic findings ranges from minor mesangial changes to focal and diffuse mesangial proliferation to crescentic glomerulonephritis. In addition to glomerular lesions, various tubulointerstitial and vascular lesions also can be seen, including tubular atrophy, interstitial fibrosis, interstitial cellular inflammation, and vascular sclerosis. However, the histopathologic criterion for IgAN is the dominance or co-dominance of IgA deposition in the mesangium. Although many other diseases also are associated with glomerular IgA deposits5, I will not consider their prognosis and treatment in this review.

42

Table 1. Clinical risk factors for progression in IgA nephropathy (IgAN) Impaired renal function at discoverya Magnitude and duration of proteinuriaa Hypertension at presentationb Older age at onset Absence of recurrent macrohematuria Persistent microscopic hematuria Male gender Obesity Hyperuricemia Hypertriglyceridemia Smoking Familial disease

Table 2. Histologic risk factors for progression in IgA nephropathy (IgAN) Glomerular Advanced glomerulosclerosisa Mesangial hypercellularity Segmental necrotizing lesions Extensive crescents (>30%) Capillary wall deposits of IgA Tubulointerstitial Marked tubulointerstitial fibrosisa Intense interstitial infiltration Arteriosclerosis/arteriolosclerosis

The magnitude and character of proteinuria are powerful clinical indicators of an adverse outcome. At present, it is believed that no sharp dividing point relates the magnitude of proteinuria to prognosis. Most investigators, however, believe that more than 1 g/day is a

43

reasonable threshold for concern11,15. Universal consensus exists that proteinuria >3 g/day is associated with a high likelihood of a subsequent progressive decline in renal function Less concordant are the data for the prognostic value of arterial hypertension at presentation. High blood pressure (>140/90 mm Hg) can be an important factor associated with a more rapid progression in adults with IgAN. However, when this clinical factor was evaluated using multivariate analysis in several studies, it appeared to be significant in some but not all

Treatment The patients with IgAN with normal renal function, minimal (<1.0 g/day) or no proteinuria, normal blood pressure, and minimal or mild lesions on renal biopsy should not be treated as benign course of patients, they should be kept under periodic review. On the other hand, patients with one or more adverse prognostic features such as hypertension, proteinuria >1 g/day, slowly progressive renal failure, and moderate to severe changes in renal biopsy histology could be candidates for one or even several therapeutic modalities. There are many approaches to treat adults with primary IgAN includes the following:

ACE inhibitors Control of blood pressure remains the cornerstone of treatment as for patients with other types of kidney disease. The therapeutic benefit of antihypertensive drugs is thought to be a putative reduction of glomerular hypertension that provides protection against glomerular injury. Because of their ability to specifically reduce intraglomerular pressure, ACE inhibitors have attracted the most attention as the ideal antihypertensive drugs for the treatment of hypertension in renal disease, including IgAN. The effectiveness of ACE inhibitors in reducing progression of renal dysfunction is posited to be a consequence of their antiproteinuric effect rather than attributable to their blood- pressure-lowering effect alone. Moreover, ACE inhibitors also might attenuate the effect of angiotensin II on renal cell growth and proliferation and might inhibit extracellular matrix component release that culminates in sclerosis.

44

Steroids Steroids were among the first agents used for the treatment of IgAN. In an early, randomized, prospective, controlled trial in adult patients with IgAN and the nephrotic syndrome, 4 months of therapy with moderate doses of oral steroids did not produce any benefit except in patients with very mild histologic lesionsAlso, Kobayashi et al reported beneficial effects of steroids given in moderate doses for periods of 1 to 3 years in patients with moderate or heavy proteinuria. In both studies, renal function was preserved in patients with an initial creatinine clearance of >70 mL/min but not in those with more severe impairment of renal function. The same authors also have demonstrated that daily steroids for 18 months in IgAN patients with normal renal function and moderate proteinuria produced a better preservation of renal function and a greater reduction in proteinuria 10 years after therapy when compared with an untreated group.

Immunosuppressive drugs There is limited experience with the use of azathioprine in the treatment of IgAN in adults. Goumenos et al published a retrospective study that used long-term azathioprine combined with low-dose prednisone in patients with progressive renal disease accompanied by proteinuria and moderate to severe histologic changes. Eighty percent of patients receiving azathioprine exhibited stable renal function, but only 36% of the group treated conservatively. There was no effect on proteinuria.. Cyclophosphamide has been used in a limited number of trials. Some of these studies used a combination of cyclophosphamide, dipyridamole, and warfarin and chiefly showed a reduction in proteinuria. Cyclosporine was tested in a randomized single-blind trial conducted by Lai, Lai, and Vallance-Owen. The drug was given for 12 weeks to 24 patients with proteinuria of at least 1.5 g/day and reduced renal function. They showed a modest reduction in proteinuria, unfortunately often accompanied by a rise in serum creatinine. These changes were reversed after cessation of the treatment. Thus, while cyclosporine is likely to display an antiproteinuric effect in IgAN, the risk of nephrotoxicity limits its usefulness. 45

Limited trials of mycophenolate mofetil (MMF) have been reported. In a few case reports and a small uncontrolled trial, MMF, 1 to 2 g/day alone or combined with steroids for several months, decreased proteinuria and stabilized serum creatinine in patients with severe IgAN

Fish oil Interest in fish oil as a treatment of IgAN has recently intensified. Fish oil is rich in -3 fatty acids, eicosapentaenoic acid (EPA), and docosahexaenoic acid (DHA). These compete with arachidonic acid and produce biologically less-effective prostaglandins and leukotrienes that might retard renal damage by decreasing glomerular and interstitial inflammation, platelet aggregation, and vasoconstriction. In the United States, a randomized controlled trial is underway comparing fish oil, alternate-day prednisolone, and ACE inhibitors in children and young adults with IgAN. The results of this trial should considerably clarify whether steroids or fish oil is better for patients at risk for progressive renal disease.

Anticoagulant and antiplatelet drugs Anticoagulant and antiplatelet therapy have long been used in the treatment of IgAN. Most of the early trials were uncontrolled and small in size.

Other treatment considerations Tonsillectomy also has been recommended for patients with IgAN, but its role remains controversial. Removal of the tonsils in patients with chronic or recurrent tonsillitis reduces serum IgA levels, circulating immune complexes, proteinuria, and episodic hematuria. Patients most likely to benefit from tonsillectomy had preserved renal function (serum creatinine <1.4 mg/dL) and mild changes on histology. These benefits were not observed in patients with more advanced renal disease. Tonsillectomy in combination with pulse steroid therapy might produce higher rates of clinical remission. However, no study to date has shown long-term preservation of renal function in patients who have undergone tonsillectomy. Tonsillectomy has not yet been

46

tested in a controlled randomized trial, so this approach should be reserved for recurrent episodes of macroscopic hematuria or relief of chronic recurrent tonsillitis.

Conclusions For the moment, no consensus exists on the best treatment of IgAN. Before making any therapeutic decisions, clinicians should keep in mind the chronic nature of the disease and the possibility of a good outcome without therapy. Treatment, therefore, should be relatively nontoxic and probably limited to patients who have reasonable evidence of a poor long-term prognosis. Hypertension should be treated aggressively, mainly with ACE inhibitors or ARBs. General agreement exists that patients with normal renal function, no or trivial proteinuria (<0.5 g/day), normotension, and minimal or mild lesions on renal biopsy should be managed conservatively with regular follow-up. It is not known whether patients with proteinuria > 0.5 g/day but <1 g/day should be treated. In my opinion, even if they are normotensive, one should attempt to lower proteinuria to <0.5 g/day by giving these patients fish oil and an ACE inhibitor or an ARB. Patients with greater proteinuria (>1 g/day), normal or mildly impaired renal function (serum creatinine <1.4 mg/dL), and mild-to-moderate renal lesions should initially receive fish oil in combination with an ACE inhibitor or ARB in an attempt to lower proteinuria to <1 g/day. If this reduction cannot be achieved, then a course of low-dose steroids on a daily or alternate-day basis for 3 to 6 months is indicated. Whether the combination of these two therapies is more effective is not clear as yet. Patients with persistent proteinuria >1 g/day, impaired renal function (serum creatinine >1.4 mg/dL), and moderate to severe histologic changes are less likely to respond to steroids. Again, these patients could be successfully treated with fish oil in combination with ACE inhibitors or ARBs, as indicated by the international experience and our own. Alternatively, azathioprine or MMF and low-dose alternate day prednisolone could be

47

considered. When contraindications to prednisolone or immunosuppressive drugs exist, low-dose warfarin combined with dipyridamole could be used. Patients with NS and minimal glomerular changes appear to respond to steroids. For nephrotic patients with more severe histologic changes, a combination of steroids with immunosuppressive drugs is indicated, as they appear to respond less favorably to steroids alone. Fish oil also might be effective in some of these patients. Patients with rapidly progressive crescentic glomerulonephritis can benefit from steroids, either pulse therapy or oral prednisolone in combination with cyclophosphamide, and sometimes supplemented by plasma exchange. High-dose intravenous immunoglobulin therapy also might be of benefit in this group. Patients with acute renal failure after a bout of macrohematuria demonstrating a paucity of crescents (<30% segmental crescents) on renal biopsy but predominantly tubulointerstitial changes seem to spontaneously heal and recover even if dialysis is required. Patients with chronic renal failure (serum creatinine > 3 mg/dL) should be treated symptomatically. Fish oil combined with an ACE inhibitor or an ARB might stabilize progression in some of these patients. For the moment, there is no specific therapy for recurrent IgAN in renal allografts. The use of ACE inhibitors might be beneficial and MMF might prevent recurrent disease.

Reference: [11], [12]

48

Acute kidney injury due to insect bite: case report Back ground: Insect stings and bites are known to cause a variety of allergic reactions and direct toxic effects. Stinging insects are classified as hymenoptera which includes apids (honey bees, Africanized bees) and vespids (wasps, yellow jackets & hornets). Hornet stings are generally followed by minor allergic reactions and rarely anaphylaxis. However multiple stings can sometimes lead to angioedema, vasculitis, encephalitis and acute renal failure. Usually acute renal failure is due to acute tubular necrosis secondary to intravascular hemolysis, rhabdomyolysis or shock .Rarely it can be due to the development of tubulo-interstitial nephritis . In this report we describe an aged male who had insectt stings and developed acute renal failure as a result of tubulo-interstitial nephritis.

Case summary: A 60-year-old British man had a history of a insect bite on the anterior aspect of his left leg resulting in cellulitis. Several days later, the patient presented to the emergency room complaining of fevers, severe itching, swelling, and blistering of the left leg. The past history was unremarkable. There was no history of asthma or use of medications. Physical examination at that time was consistent with a bullous cellulitis of the left leg. Laboratory investigations revealed: white blood cell count (WBC), 10.3 differential: 54 neutrophils, 15 lymphocytes, and 31 eosinophils. The platelet count was 225,000/mm3. A blood chemistry profile was within normal limits. Blood cultures and stool examination for ova and parasites were negative. Leg X-ray: no boney abnormality Chest X-ray- clear C/S: green sputum,USS of left leg: no evidence of DVT (Distal calf vessels not arrested) According to the patient statement he had leg erythema with swelling for the last 2 months which was increased overtime. He reported first leg swelling and redness appeared 29/6/07 then it turned to dark color with swollen persist. He also added some blister lifting off from the infected area. But patient clinically looked well.

49

Patient also has history of insect bite on ankle and that time he was treated with anti histamine. Then he developed increase blood pressure for few years and take atenolol. Patient mentioned that he has had chronic right ankle pain, hip and shoulder pain with arthritis.

Case report: Name: R D S Age 60 Presenting complains: Pain in the let lower leg Swelling of lower left leg Redness with some blister is lifting off from affected leg

HPI: According to patient statement he was leg redness for 2 months swelling increased as day passes. He is reported first leg swelling and redness appeared 29/6.07 then it become dark swollen. He also added some blister lifting off from the infected area. But patient clinically well look Patient also has history of insect bite on ankle and that time he was treated with anti histamine. Then he developed increase blood pressure for few years and take atenolol. Patient tells me he has had chronic right ankle pain, hip and shoulder pain with arthritis

PMH: Renal failure, insect bite, HTN, Arthritis Personal history: Ex smoker, Medication: Metaclopramide 5mg tds Salbutamol 5mg od Nefoman 30mg od 50

Quinine sulphate 300mg od Fluoxetine20mg od Sodium bicarbonate 50mg O/E: Leg erythoma extending to just below knee Mild wheeze Good foot pulse Leg look like cellulitis Splinter haemorrage on hand nail P/A: Soft non tender epigastrium Vascular bruit CVS: Heart sound normal, JVP (-) Right ankle odema RS Very few basal crepitation

Investigation: 06/07/07 HB 11.6 WCC 19 CRP 370 NA 139 K 4.5 UREA 28.7 CR 701 HCO3 28 CL 105 Leg X-ray: no boney abnormality Chest X-ray- clear C/S: green sputum USS of left leg: no evidence of DVT (Distal calf vessels not arrested)

51

Discussion: This case demonstrates that multiple bee stings may cause rhabdomyolysis and hemolysis with consequent ATN. Components of venom responsible include toxic surface-active polypeptides (mellitin and apamin), enzymes (phospholipase A 2 and hyaluronidase) and low molecular weight agents (histamine and aminoacids). Mellitin and phospholipase are important components causing rhabdomyolysis following a toxic action on striated muscles and also act on red cell membrane and provoke hemolysis. The elevated levels of enzymes CPK and aspartate-aminotransferase suggest the existence of rhabdomyolysis and hemolysis is suggested by anemia, unconjugated hyperbilirubinemia, reticulocytosis, increased serum LDH and hemoglobinuria. Rhabdomyolysis and hemolysis can induce ARF, particularly in hypovolemic or acidotic individuals. It has been postulated that myoglobin and hemoglobin released from muscle or red blood cells cause ARF by toxic effects on tubule epithelial cells or by inducing intratubular cast formation. Hypovolemia or acidosis may contribute to pathogenesis of ARF in this setting by promoting intratubular cast formation. In addition, both hemoglobin and myoglobin are potent inhibitors of nitric oxide bioactivity and may trigger intrarenal vasoconstriction and ischemia in-patient with borderline renal hypoperfusion. The primary therapeutic goal is to prevent the factors that cause ARF, i.e. volume depletion, tubular obstruction, aciduria and free radical release. Patients are administered saline for intravascular volume expansion and sodium bicarbonate for urine alkalization (to urine pH level above 7). The ideal fluid regimen for patients with rhabdomyolysis consists of half isotonic saline (0.45%, or 77 mmol/L sodium), to which 75 mmol/L of sodium bicarbonate is added. Once overt renal failure has developed, the only reliable therapeutic modality is extracorporeal blood purification. The treatment of severe ARF following Africanized bee stings with repeated hemodialysis, hemofiltration, or peritoneal dialysis has been described. Exchange transfusion or plasmaphresis has been found useful because it acts through a direct effect of reduction of the massive circulating

52

venom or removal of the circulating mediators of inflammation caused by the venom itself. Taking into account the potentially lethal risks of plasmapheresis, this should be reserved for the treatment of life-threatening multisystem organ failure due to stings of members of the order Hymenoptera (honeybee, bumblebee, wasp, hornet, yellow jacket). In conclusion, acute renal failure after bee stings is probably due to pigment nephropathy associated with hypovolemia. Early recognition of this syndrome is crucial to the successful management of these patients.

Reference: [13]

53

Diabetic nephropathy and hypertension: case report Back ground: Hypertension and diabetes mellitus are interrelated diseases, which, if untreated, strongly predispose to atherosclerotic cardiovascular disease. Lifestyle and genetic factors are important in the genesis of both conditions The prevalence of hypertension and type II diabetes, or non-insulin-dependent diabetes mellitus (NIDDM), increases with age. Hypertension occurs more frequently in persons with type I diabetes, or insulindependent diabetes mellitus (IDDM), than in those with type II diabetes after adjustment for age. Essential hypertension accounts for the majority of hypertension in persons with diabetes, Particularly in those with type II diabetes, who constitute more than 90 percent of those with a dual diagnosis of diabetes and hypertension. Diabetic nephropathy, which occurs after 15 years of diabetes in one of three persons with type I diabetes and one of five persons with type II diabetes, appears to be an important cause of hypertension. Hypertension can exhibit several kinds of pathophysiology in patients with diabetes. Hypertension associated with diabetic nephropathy is a form of renal hypertension with sodium and fluid retention, increased peripheral vascular resistance, and increased cardiac output, especially if anemia is present. Isolated systolic hypertension can occur at any age and is considerably more common in persons with diabetes than in those without diabetes.Hypertension contributes to the leading causes of morbidity and mortality in persons with diabetes, including coronary heart disease, stroke, peripheral vascular disease, lower extremity amputations, and end-stage renal disease. Hypertension also contributes to diabetic retinopathy

case summary: 71 years, british man admitted in Northern General Hospital with complain of difficult in micturation, Cough with sputum for 6 days,

54

SOB for 2 days, Chest pain 2 or 3 days patient admitted for renal biopsy and waiting for biopsy report. According to the pt he was alright then after 4 days he devolved shortness of breath which was associated with chest pain and followed by cough with sputum. Also he is complaing of nausea and vomiting and anorexia for last 7 or 8 days. Now patient has had catheterization in situ for 2 days as he suffering from oliguria. Patient also had past history of MI, HTN, Asthma, Pleural effusion, DM, Epilepsy Name: B B Age 71 Presenting complains: Currently feeling well, difficult in micturation, Cough with sputum for 6 days, SOB for 2 days, Chest pain 2 or 3 days Patient has had catheterization in situ. HPI: patient admitted for renal biopsy and waiting for biopsy report. According to the pt he was alright then after 4 days he devolved shortness of breath which was associated with chest pain and followed by cough with sputum. Also he is complaing of nausea and vomiting and anorexia for last 7 or 8 days. Now patient has had catheterization in situ for 2 days as he suffering from oliguria.

PMH: MI, HTN, Asthma, Pleural effusion, DM, Epilepsy Personal history: live with wife, ex smoker.

Drug history: RENAGEL 800mg bd Alfacalcidol 0.25mcg od Mixtard 30 u (m) 45u (e) 55

Ciprofloxacin 250mg bd for 7 days Frusemide 160mg bd Metalozone 5mg bd Lopramide 2mg od Paracetamol 1gm sos NSAID, Beclomethasone BD O/E: Leg oedema and sacral oedema present JVP is raised. RS: Chest clear CVS: Heart sound is normal P/A: suprapubic tender, soft and non tender abdomen.

Investigation: 20/07/07 HB 11.6 WCC 8.6 PLTS 387000 NA 132 K 4.3 HCO3 23 UREA 24.2 CR 434 CA 2.28 POA 1.89 TOTAL PROTEIN 77 ALBUMIN 31 AST 19 BILIRUBIN 10 US: 5/7/7- No hydronephrosis

56

Discussion: A medical history should include family history of hypertension, diabetes, renal disease and CVD; patient history of CVD, cerebrovascular disease, renal disease, and retinopathy; known duration and levels of elevated blood pressure and blood glucose; results and side effects of previous antihypertensive and hypoglycemic therapy; use of drugs that may influence blood pressure or diabetes; history of weight gain or loss, proteinuria, sodium intake, other dietary factors, exercise habits, and alcohol use; symptoms suggesting secondary hypertension; psychosocial and environmental factors (e.g., emotional stress, cultural food practices, economic status) that may influence blood pressure or blood glucose control; other cardiovascular risk factors including obesity, smoking, and hyperlipidemia; and date of last eye examination. With regard to diabetes, information should be sought about the history of polyuria, polydipsia, polyphagia, fatigue, blurred vision, hypoglycemic reactions, sexual dysfunction, paresthesia or other signs of peripheral neuropathy in the extremities, and leg and foot ulcers.Diabetes With Renal (Parenchymal) Hypertension A persistent increase in albumin excretion rates on at least three occasions and/or a progressive decline in renal function suggest a diagnosis of renal (parenchymal) disease. Hypertension in patients with diabetic nephropathy is manifested by sodium and water retention (extracellular fluid volume expansion), increased cardiac output, and increased peripheral vascular resistance. Although these mechanisms do not differ from those usually encountered in patients with hypertension who do not have diabetes, special consideration must be given in choosing antihypertensive agents for patients with diabetes and renal disease. Others have included persons with diabetes who do not have hypertension and have used microalbuminuria as a surrogate for alterations in renal function. Some, but not all, studies with calcium antagonists have suggested a reduction in albumin excretion, and at least one study has suggested an additive benefit when an ACE inhibitor and a calcium antagonist are used together.

Reference: [14]

57

P-ANCA positive vasculitis: case report Background: The perinuclear antineutrophilic cytoplasmic antibody (p-ANCA) is closely associated with rapidly progressing glomerulonephritis, microscopic polyangiitis, and allergic granulomatous angiitis. While mononeuropathy due to vasculitis is a well- known neurological manifestation of these conditions, manifestations involving the central nervous system (CNS) have rarely been reported.

Case summary: A 78 year old man admitted in northern general hospital to make AVF for dialysis purpose but unfortunately failure of left brachiochephalic AVF happened. The patient of Pakistani origin that has been refers to the clinic for abnormal renal function. He was recently admitted for episode of angina, P-ANCA positive, 24 hrs urine excretion 1.3 gm, Creatinine Clearence 30 l/min, suggesting renal failure. Complement level is normal. He had a right eye cataract removal. He was well himself and fairly asymptomatic. He occasionally complains of angina on exertion. His blood pressure was well with in normal range. Investigation so far showed a significantly impair renal function with serum creatinine show below and a GFR of around 20ml/in. this is most likely due to long standing HTN, Ischemic nephropathy. There is no haematuria to suggest a glomerular nephritis. However as mention in the previous level his P-ANCA serology has been slightly rise. We are keep an eye on his immunology, advice for 24 hrs urine collection to estimate proteinuria as has +++ albumin in Dipstick analysis. Over the last 2-3 is a reflection of good blood pressure control, possibly ACEi and the fact that he is a non smoker. I have also stated him in a statin even his serum cholesterol is with in normal range

Case report: Name: M M Age: 78 years 58

Present Complain: Failure of left brachiochephalic AVF (Exploration of left radio cephalic AVS done)

HPI: 78 yrs, Pakistani origin that has been refers to our clinic for abnormal renal function. He was recently admitted for episode of angina, P-ANCA positive, 24 hrs urine excretion 1.3 gm, Creatinine Clearance 30 l/min, suggesting renal failure. Complement level is normal. He had a right eye cataract removal. He was well himself and fairly asymptomatic. He occasionally complains of angina on exertion. His blood pressure was well with in normal range. Investigation so far showed a significantly impair renal function with serum creatinine show below and a GFR of around 20ml/in. this is most likely due to long standing HTN, Ischemic nephropathy. There is no haematuria to suggest a glomerular nephritis. However as mention in the previous level his P-ANCA serology has been slightly rise. We are keep an eye on his immunology, advice for 24 hrs urine collection to estimate proteinuria as has +++ albumin in Dipstick analysis. Over the last 2-3 is a reflection of good blood pressure control, possibly ACEi and the fact that he is a non smoker. I have also stated him in a statin even his serum cholesterol is with in normal range.

PMH: CRF, HTN, P-ANCA positive, IHD Personal History: Ex smoker

Drugs history: Currentl he is taking Na Bicarb Norton – 500mg cap Glyceryl trinitrate 400 micrograms Quinine Sulfate 300 mg Furosemide 40mg

59

Isosorbide mononitrate 60mg Simvastatin 20mg Ranitidine 150 mg Amlodipine 5 mg Aspirin 75 mg O/E: Failure AVF left hand P/A: Soft non tender abdomen CVS: Heart sound normal, JVP (-) RS: Clear chest wall

Investigation: 08/08/07 HB 11.3 HCT 0.35 WCC 5.3 PLTS 168000 NA 139 K 4.9 UREA 37.6 CR 474 HCO3 17 CA 2.21 PO4 2.17 ALK.PO4 105 60

TOTAL PROTEIN 61 ALBUMIN 33 The date of investigate is 10/07/2007 Urea-30.3 Creatinine- 506

Discussion: The pathogenesis of ANCA-related vasculitis is very complicated, and several mechanisms are considered to be involved together in its development. In the present case, Damage to the endothelium may lead to an increase in vascular permeability and extravasation which can break the blood-brain barrier and eventually result in uncontrollable vasogenic edema. In this regard, we feel that the dilatation of intracranial vessels seen in the present case especially that of the peripheral small arteries must be significant.Vasospasm has often been seen in RPLS associated with hypertension, but the MRA findings suggest vasodilatation in the present case. This disease entity represents a relatively new concept and its pathogenesis remains to be determined, though coronary artery spasms, myocarditis, an over-response to catecholamine and stress have been postulated as causes. The present case suggests that it can also be associated with pANCA-positive vasculitis. The treatment of renal limited systemic vasculitis usually involves a combination of cytotoxic drugs and steroids. As shown by randomised prospective controlled trial, plasmapheresis may be of additional benefit for the management of patients with renal involvement severe enough to require dialysis support. Recently, growing evidence has suggested that autoantibodies to neutrophil cytoplasm (ANCA) may play a role in the pathogenesis of the primary vasculitides by promoting neutrophil mediated endothelial cell cytotoxicity. This has led to new strategies for treatment based on firstly, the use of semi-specific immunoabsorption (IA) devices to remove circulating autoantibodies, and secondly, the use of 'Humanised' monoclonal antibodies (MAbs) with specificity for lymphocytes, particularly T lymphocytes. We have treated four patients, two with ANCA specificity for proteinase 3 (PR3), and two with specificity for myeloperoxidase (MPO).

61

Semi-specific IA was carried out by plasmapheresis through extracorporeal online devices, using L tryptophan as the immobilised immunoabsorbant. Of the four patients who received IA, three showed substantial depletion in ANCA titres and resolution of clinical symptoms. The MAbs were subsequently used to attempt to obtain long-term control of ANCA synthesis. These results suggest that an optimal strategy for treatment of systemic vasculitis might consist of specific IA, using immobilised ANCA antigens to deplete circulating vasculotoxic antibodies, combined with MAb therapy to restore immune homeostasis. Reference: [15], [16]

62

Assessment and management of comorbid diseases in CKD: case report

Background Chronic kidney disease (CKD) is common and can be found in up to 23% of patients with diabetes. The recommended hemoglobin A1c goal for these patients is also < 7.0%. Medication therapy for diabetes may require dose adjustments or may be contraindicated in patients with CKD. Assessment and management of comorbid diseases, including hypertension, hyperlipidemia, anemia, hyperphosphatemia, and hyperparathyroidism, is important in the care of patients with diabetes and CKD. Multidisciplinary care may provide the optimal system for maximizing care of these complex patients.

Case summary: A 84 years old British man get admission in Northern General Hospital with complain of pain on upper abdomen for 4/5 days, loss of appetite, nausea, vomiting for 4-5 days. Patient was seen 1 week back in gastric clinic for investigation of his microcytic anemia. He slept well overnight but his morning he has vomited for 5 times and had an episode of diarrhea as well. He has epigastric pain which was mild to moderate in intensity. Blood test in the OPD showed K increased 6.4 mmol/l Patient denies any chest pain on his discomfort or shortness of breath. No haemoptysis, malena, bleeding PR noticed. He also denies any urinary symptoms what so ever. Also he had past history of MI-.1992 Angina for more than 15 yrs ,CKD stage Diabetes for 20 years

Case report: Patient name: S G Age: 84 yrs Sex: male 63

Present complain: Pain on upper abdomen for 4/5 days Loss of appetite, nausea and vomiting for 4-5 days Bodyache for 1 week Also informs me increased potassium level.

HOPI: Patient was seen 1 week back in gastric clinic for investigation of his microcytic anaemia. He slept well overnight but his morning he has vomited for 5 times and had an episode of diarrhea as well. He has epigastric pain which was mild to moderate in intensity. Blood test in the OPD showed K increased 6.4 mmol/l Patient denies any chest pain on his discomfort or shortness of breath. No haemoptysis, malena, bleeding PR noticed. He also denies any urinary symptoms what so ever.

PMH: MI-.1992 Angina for more than 15 yrs CKD stage 3 Diabeties for 20 yrs Social history: Non smoker Non alcoholic

Drug history Omeprazole 10mg Aspirin 150 mg Gliclazide 160 mg GTN

64

Lisinopril 2.5 mg Metformin 1 gm On examination Patient is well co-operative oriented with time, place and person CVS: BP- 150/80 Pulse 96 b/min JVP- not raised HS: normal No pedal odema

RS: Decreased air entry in both lungs No cough sputum, wheeze, Crepitation, O2 saturation 94% on air GIT: Soft non tender No sign of viseromegaly or peritonitis. Bowel sound is audible CNS GCS 15 Cranial nerve intact

Investigation Hb 9.7

65

Na 138 K 6.4 Ca 2.44 PO4 –1.01 TSH- 1.14 B12- 377 Ferritin-32.0 HbAIC- 6.7% HCO3- 18

Diagnosis: Normocytic anemia MI Type 2 DM Impression Sepsis

Discussion: Management of cardiovascular comorbid Disease Hypertension: Hypertension is commonly found in patients with DKD and is diagnosed by a blood pressure measurement > 130/80 mmHg, as defined by the Seventh Report of the Joint National Committee on the Prevention, Detection, Evaluation, and Treatment of High Blood Pressure. Prevalence of hypertension is estimated to range from 30 to 96%, with higher prevalence found to be associated with greater levels of proteinuria Hypertension that is not controlled leads to a higher risk of cardiovascular events including death, increasing proteinuria, and progression of kidney disease. The cornerstone of medical therapy for hypertension, especially in patients with CKD, is treatment with angiotensin-

66

converting enzyme (ACE) inhibitors and angiotensin receptor blockers (ARBs). ACE inhibitor or ARB therapy has been demonstrated to slow the progression of proteinuria in patients with either type 1 or type 2 diabetes and microalbuminuria; however, no randomized clinical trials have shown impact on development of more advanced CKD or mortality in this population. Hyperlipidemia Cardiovascular disease is common in patients with both diabetes and kidney disease. Risk factor modification, including management of dyslipidemia, is a key component of care for this patient population. Lipid levels should be measured annually, with a target LDL cholesterol level < 100 mg/dl for patients with CKD stages 1-4. In a study of nearly 20,000 patients, those with diabetes and CKD who received pravastatin compared to placebo were found to have a 25% relative risk reduction of cardiovascular disease events. Consideration of dosing of drugs for dyslipidemia therapy must also take into account severity of kidney disease. No dosage adjustments are required for bile acid sequestrants, niacin, ezetimibe, atorvastatin, or pravastatin. Reduced dosing of fibric acid derivatives, fluvastatin, lovastatin, rosuvastatin, and simvastatin should be considered in patients with Stage 4 or 5 CKD Complication: Anemia in CKD is defined as a hemoglobin value < 13 g/dl for males and < 12 g/dl for females, and annual evaluation is recommended. Correction of anemia to levels of 11-12 g/dl in dialysis patients has been associated with improved quality of life, fewer hospitalizations, and a lower risk of mortality; however, studies in patients with CKD (pre-dialysis) are lacking. Two recent clinical trials, the Cardiovascular Risk Reduction in Early Anemia Treatment With Epoetin Beta study and the Correction of Hemoglobin and Outcomes in Renal Insufficiency study, suggested that hemoglobin levels > 12 g/dl did not improve measures of quality of life and may increase the risk of cardiovascular events. Additional clinical studies are ongoing to try to provide guidance in this complex area.Abnormal calcium and phosphorus metabolism may also be present in patients with CKD. These measures, along with measurement of intact-parathyroid hormone (i-PTH),

67

may identify bone disease related to CKD. Bone disease may lead to poor bone structure resulting from high or low turnover, and this may result in a higher risk of fracture. Frequency of measurement of calcium, phosphorus, and i-PTH is described in The target serum phosphorus goal is < 5.5 mg/dl in patients with Stage 5 CKD and < 4.6 mg/dl in Stage 3-4 CKD. In Conclusion: CKD and diabetes are common diseases that affect a large proportion of the population. Depending on the severity of the CKD, drug regimens, including those for glycemic control, and dietary intake may require adjustments . Aggressive identification and treatment of risk factors for cardiovascular disease as well as complications of CKD are recommended given the very high risk of adverse cardiovascular events in patients with both diabetes and CKD.. Comments: Chronic kidney disease (CKD) is common and can be found in up to 23% of patients with diabetes.The recommended hemoglobin A1c goal for these patients is also <7.0%. Medication therapy for diabetes may require dose adjustments or may be contraindicated in patients with CKD. Assessment and management of comorbid diseases, including hypertension, hyperlipidemia, anemia, hyperphosphatemia, and hyperparathyroidism, is important in the care of patients with diabetes and CKD..

Reference: [17], [18], [19]

68

Wegener’s granulomatosis: case report

Background Wegener's granulomatosis is one of the pauci-immune small vessel vasculitides. It classically presents with the triad of upper and lower respiratory tract granulomas and necrotising focal segmental glomerulonephritis. It is associated with the presence in the serum of autoantibodies against components of neutrophil cytoplasm–antineutrophil cytoplasmic autoantibodies (ANCA). The illness can develop at any age but is more common in patients in their 50s and 60s and in men. The incidence of vasculitis is increasing, with about 10-20 people per million affected. We present a case that in retrospect had many clues at the initial time of admission, but it took five months and six different hospital teams to make the diagnosis.

Case Summary A 71 year old woman, admitted to Northern General hospital at Vickers 2 with a complain of shortness of breath. According to the patient she was alright 5 yrs back then she was diagnosed as a case of Wegners Granulomatosis. WG has been well controlled and relatively symptom free but over last 6-8 months she has been getting sinus symptoms and hearing loss. But now for last 2-3 days the symptoms are very much problematic. As she told me that she cant walk too long as she developed shortness of breath and tiredness. As by saying that yesterday evening she was unbable to go up stair one at a time, she took about an hour to do so and needs her husband assistance to get up and dressed this morning, feeling so lethargic and weak. She feel hot and sweaty does not feel unduly well in herself if breathing was better. She also says that she has mild pain at the right breast for 3 weeks or so but was relieved by sitting forward. She have not take any analgesic for that. She does not give me any history of urinary abnormality like haematuria and increased urinary frequency, nausea, vomiting, abdominal pain or indigestion

69

Case Report Name: E C Age: 71 yrs Presenting complain: Shortness of breath for 4 days

HOPI: According to the patient she was alright 5 yrs back then she was diagnosed as a case of Wegners Granulomatosis. WG has been well controlled and relatively symptom free but over last 6-8 months she has been getting sinus symptoms and hearing loss. But now for last 2-3 days the symptoms are very much problematic. As she told me that she cant walk too long as she developed shortness of breath and tiredness. As by saying that yesterday evening she was unbable to go up stair one at a time, she took about an hour to do so and needs her husband assistance to get up and dressed this morning, feeling so lethargic and weak. She also noticed blood in spit she think that blood causes from post nasal site. She feel hot and sweaty does not feel unduly well in herself if breathing was better. She also says that she has mild pain at the right breast for 3 weeks or so but was relieved by sitting forward. She have not take any analgesic for that. She does not give me any history of urinary abnormality like haematuria and increased urinary frequency, nausea, vomiting, abdominal pain or indigestion

PMH: WG PD in past but stopped because renal function improved. Hypothyroidism after partial thyroidectomy

Drug history: Prednisolone 15mg Azathioprime 150mg

70

Septrin 480mg Pravastatin 20mg One alpha 250mg Levothyroxine 75mg Allegies history: NAD Social history: Retired Never smoked No alcoholic Live with husband and sons

On examination Co operatiove well oriented comfortable looking CVS JVP- not raised BP-130/90 Pulse 86 b/min Warm peripheries Slightly pitting oedema left foot Respiratory system: Bilateral crepts R>L Bronchial breathing Wheeze Decreased air entry on right mild zone R/R- 20 br/min Sat 100% on 40 O2 (92% on air)

71

GIT: Soft no tender Bowel sound audible Eyes Swollen eyelids Left eye may be conjunctivitis Eye movement normal According to her vision is normal

Investigation Urine dipstick: Gl- negative, Ketones-negative, Blood ++ ve, Protien: +++ ve Leucocyte trace, nitrate- negative Hb- 9.3, WCC-13.1, MCV-96.3, N-12.3 increased, L-0.4 decreased, ESR- 108 PT- 16, APTT- 40.2, PTH-123 Cr- 209 Urea 20.3 Alb- 25 AST-27 Ca- 2.71 Chest X-ray: Dense opaciy right mild zone Well defined opacity left mild zone Right hemidiaphragm elevated Impression Right middle lobe pneumonia

72

Hypercalcemia Renal impairement Progression of WG

Discussion Vasculitis can be categorised by the size of vessel affected (small, medium, or large). Wegener's granulomatosis is a small vessel vasculitis classically involving the upper and lower respiratory tracts and kidneys. A limited form involving the upper respiratory tract has been described in a quarter of cases. The most common presenting symptoms include most upper airway respiratory symptoms, but particularly rhinorrhoea, oral ulcers, nasal discharge, polyarthralgias, myalgias, and sinus pain. Lower airway complaints such as cough, dyspnoea, haemoptysis, and pleuritic pain are also early features. More rarely, Wegener's granulomatosis can present with tumour-like masses distant from the lung. The renal disease often presents with haematuria, red cell casts, proteinuria, and renal failure. Other systems that may be involved include joints, eyes, skin, nervous system, heart, and gastrointestinal tract. Serological testing for ANCA is a useful diagnostic aid for small vessel vasculitis,2 although caution is needed as the sensitivity may be as low as 66% when the disease is not severe.3 The diagnosis should be suggested from clinical as well as laboratory findings and where possible confirmed with a tissue biopsy. Treatment in Wegener's granulomatosis can be life saving. It is usual to obtain a tissue diagnosis before starting treatment, but if the patient is too unwell, as in this case, treatment should not be delayed. Untreated Wegener's granulomatosis has a poor prognosis, with most patients dying within two years.1 With cytotoxic treatment, this has improved, and the eight year survival is about 80%.1 Up to 90% of patients respond to cyclophosphamide, and three quarters have a complete remission,4 but pulmonary haemorrhage remains a potentially life threatening complication. Induction treatment is usually given as combined treatment with cyclophosphamide and corticosteroids. Use of corticosteroids alone is associated with a higher relapse rate.5 The specifics of induction regimens vary with the drugs used, doses, routes of administration, and duration. Patients with severe disease are treated with either plasma exchange or pulsed intravenous

73

methylprednisolone. The results of a randomised study comparing these two treatments when used as an additional early treatment in patients with severe kidney damage are awaited (MEPEX trial, European Vasculitis Study Group). Recovery from dialysis dependence is common; 55% to 90% of patients recover their renal function, and 40% to 70% manage without dialysis for three years or more. This case illustrates the importance of considering vasculitis in differential diagnosis of patients with multi-system disease. The clinical manifestations of small vessel vasculitis can be very varied because they are influenced by the sites of involvement and disease activity. ANCA testing is highly sensitive for detecting pauci-immune small vessel vasculitis, although its specificity depends on the patient population tested. Early diagnosis allows appropriate treatment to be started before serious and potentially life threatening complications have developed.

Reference: [20]

74

Pulmonary hypertension in patients with ESRD on HD: case report

Background Kidney transplantation is now recognized as the treatment of choice for patients with chronic renal failure. Despite the extension of indications to patients suffering severe hypertension, ischemic heart disease, and chronic heart failure, the worldwide results are superb. However, perioperative cardiac complications occur in 6% to 10% of transplanted patients. Aggressive intraoperative volume expansion is still recommended to maximize graft functional recovery (up to 30 mL/kg/h, central venous pressure [CVP] > 15 mm Hg), but patients with preexistent cardiac disease or poor myocardial function are exposed to the risk of fluid overload, acute respiratory failure, and prolonged ventilationAge above 50 years was the only significant risk factor. Supranormal volume loading is probably not always warranted in kidney transplantation.

Case summary: A 51 years old woman, came in Northern General Hospital with complain of shortness of breath for 3 weeks strangery for 1 week. Patient came through sorby room 1 with complaint of dyspnea or shortness of breath for 2 weeks while doing some work. She also has got muscle tenderness which gradually increased intensity by walking but got relieved by taking rest. Urine output was reduced for last 10days and also has strangury but no haematuria. She also noticed increased leg swelling. Lower abdominal wall edema also complaint of pain at biopsy performed. He had past history of NIDDM for 12 yrs MCD for since 1987, Cadaveric renal transplant on 1993, Renal biopsy on 1999 Asthma and HTN for 10 yrs, He also history of sleep apnoea

Case Report Name: R J K Age: 51 yrs 75

Presenting complain: Shortness of breath for 3 weeks Bodyache for 1 week Strangery for 1 week

HOPC: Patient came through sorby room 1 with complaint of dyspnea or shortness of breath for 2 weeks while doing some work. She also has got muscle tenderness which gradually increased intensity by walking but got relieved by taking rest. Urine output was reduced for last 10days and also has strangury but no haematuria. She also noticed increased leg swelling. Lower abdominal wall edema also complaint of pain at biopsy performed.

PMH: NIDDM for 12 yrs MCD for since 1987 Increased BMI Cadaveric renal transplant on 1993 Renal biopsy on 1999 Asthma and HTN for 10 yrs He also history of sleep apnoea

On examination Patient is well oriented and co-operative BP-163/90 mm of Hg HR-96 b/min R/R- 18 br/min SpO2- 88% on air 94% with 2 liters O2

76

CVS JVP increased Sacral edema positive Respiratory system Few crackles on right side to mid zone No wheeze Good air entry to bases Abdomen Soft and non tender No redness around groin Right leg biopsy site 1.5 cm densely non healing size necrotic pussy discharge

Investigation Hb- 8.4 WCC- 9.2 Platelets- 348 Ca- 0.98 Cl- 95 ECG- normal Chest X-ray Huge cardiomegaly with biventricular enlargement. Interstitial edema with edema with evidence of upward diversion of pulmonary vessels Impression Fluid overload failing renal transplant No healing influenced biopsy site

77

DISCUSSIONS: Various contributing factors are involved in the pathogenesis of pulmonary hypertension in patients with ESRD on HD. Volume status, diastolic dysfunction, pulmonary vascular disease, and high cardiac output from a left-to-right shunt across the arteriovenous fistula, can all contribute to pulmonary hypertension either singly or in combination. Delineating these 4 etiologies can be difficult without invasive hemodynamics. Our patient had evidence of diastolic dysfunction and fluid overload at initial catheterization as suggested by the elevated wedge pressure. However, the fall in pulmonary vascular resistance by 64% with marked improvment in cardiac index is strongly suggestive of pulmonary vascular disease as the primary underlying etiology for her pulmonary hypertension.

CONCLUSION: The reversal of pulmonary hypertension with dual endothelin-receptor antagonist in our patient supports the hypothesis that in patients with ESRD on HD, there is an alteration in endothelium-mediated vasodilators causing vasoconstriction and pulmonary hypertension. The pharmacokinetics and safety of bosentan in ESRD has already been demonstrated. This is the first reported case of improvement of pulmonary hypertension associated with ESRD on bosentan therapy.

Reference: [21]

78

Risk of mortality for ESRD patients on dialysis: case report

Background Patient survival after renal transplantation (RT) is markedly better than that seen with either haemodialysis (HD) or peritoneal dialysis. Transplantation is also associated with a better quality of life and a higher degree of rehabilitation .Some of the benefits associated with transplantation are related to patient selection, since dialysis patients with the most serious co-morbid conditions are not accepted for the transplant waiting list and hence remain in the dialysis group. Some previous comparative studies of dialysis vs renal transplant survival have considered that the transplant group is favoured by inclusion of these high-risk patients in the dialysis group. The present study uses the Cox proportional hazard regression model to compare the relative risk of mortality for ESRD patients on dialysis vs after transplantation. Analysis was adjusted for the variables that were significantly and independently related with mortality

Case summary: A 39 years old man get admission in northern general hospital with complain of shortness of breath and weekness. According to patient he was alright 1 week back then he develops dyspnea which was associated with generalized weakness and bodyache also followed by nausea, vomiting and anorexia for 3-4 days. Patient is diagnosed as ESRF on hemodialysis since 2003 He had past history of ESRF on HD since 2003, Pauci immune necrotizing GN feb 2003, HTN, Obese BMI 38, Previous left DVT, Type 2 DM, Left AVF made on 9/5/03, Cadaveric renal transplant, H/O fracture collar bone and finger

Case report: Name: S M E Sex: male DOB: 5/11/68 79

Present complain Shortness of breath for 1 week Generalized weakness for 4 days Nausea vomiting for 3-4 days

HOPI: According to patient he was alright 1 week back then he develops dyspnea which was associated with generalized weakness and bodyache also followed by nausea, vomiting and anorexia for 3-4 days. Patient is diagnosed as ESRF on hemodialysis since 2003

PMH: ESRF on HD since 2003 Pauci immune necrotizing GN feb 2003 HTN Obese BMI 38 Previous left DVT Type 2 DM Left AVF made on 9/5/03 Cadaveric renal transplant H/O fracture collar bone and finger Active bleeding piles 8/10/03 Social history: Lives with parent Unmarried Ex manager in computer firm Never smoked Take alcohol in moderation

Drug history:

80

Venofer 100mg Alfacalcidol 1 mcg Gliclazide 40 mg Neorecormon 4000 i.u Clonazepam 500mg Aforvastatin 40 mg Lactulose + senna Clopidrogel 75 mg Basiliximab 20 mg MMF from 500mg-750 mg

On examination Patient is well oriented with time place and person and is co-operative BP: 160/90 mm of Hg Pulses: 86 b/min R/R: 20 br/min CVS: Heart sound normal JVP not raised Respiratory: Decreased air entry right lung and left lung Mild crepitation on right middle lobe Also crepitation on left side as well Abdominal Soft and non tender Bowel sound audible

81

Investigation Hb: 10.5 WCC:12.1 Platelates: 189 MCV: 87.3 Urea: 10.2 Cr- 186 PO4- 1.3 Chest X-ray Shows cardiomegaly Small left plural effusion

Discussion Several studies have examined survival rates for ESRD patients and compared results of patients treated by dialysis vs patients treated by transplantation. However, many of these studies did not consider patient selection bias or time-to-treatment bias . The dialysis treatment before transplantation of most of the transplant patients was carried out in other dialysis centres. On the other hand, most of the patients who lost their transplants restarted dialysis in other dialysis centres. The importance of the co-morbidity added during dialysis treatment in RTR survival has been evaluated by the relationship between RTR survival and the length of time on dialysis before transplantation. It has been reported that increased time on dialysis prior to renal transplantation is associated with decreased survival of RTR .. The knowledge of the different outcomes in haemodialysis and after transplantation between diabetic and non-diabetic patients may be useful to health-care professionals when advising patients that they would be acceptable as transplant candidates. Diabetic ESRD patients would have a high long-term benefit when they receive a successful transplant, after a period of higher risk. The benefits for nondiabetic patients are less evident: transplantation would offer little additional survival benefit on average. Moreover, the relative scarcity of cadaveric organs for the high

82

demand and the increasing frequency of diabetes in the new ESRD patients must be considered. Diabetes is now the most common cause of new patients requiring renal replacement therapy, accounting for as many as 37% of the cases in the United States and about 15 or 20% in Europe, Latin America and Japan . These findings about survival need to be put into perspective, with the consideration of a better quality of life observed in transplant recipients compared with dialysis patients , although the quality-of-life results should also be adjusted to the significant risk factors for a better comparison Reference: [9]

83

ADPKD: case report

Background Shortage of cadaver kidney donors has prompted occasional use of abnormal kidneys such as those affected by autosomal dominant polycystic kidney disease (ADPKD) with preserved renal function. ADPKD is a hereditary disorder characterized by slow progressive deterioration of renal function due to cystic changes. Upon serious consideration of quality-of-life improvement with a functioning kidney graft, there is justification for use of cadaveric polycystic ADPKD kidneys when the intervals between progression to end-stage renal disease and graft survival period are compared. We report our experience of transplantation of two ADPKD kidneys with normal graft function.

Case summary: A 46 years old man get admitted in Nothern General Hospital with complain of Fever with rigors for 1 week, Nausea, vomiting for 3-4 days, Anorexia for 10 days Constipation for 2 days. According to patient he was alright 2 weeks back then he develops fever with rigors with sweating also associated with nausea, vomiting and anorexia also has constipation as well. He takes paracetamol for fever which works. He says that he had mild headache and lethargic. He had no history of dyssuria, haematuria or uregency. He had a cadaveric renal transplant in 3/1/2000 due to ADPKD with renal failure.

Case report: Name: Rodgers David Age :46 yrs Sex : male NGH no: 600820 Presenting complain Fever with rigors for 1 week Nausea, vomiting for 3-4 days

84

Anorexia for 10 days Constipation for 2 days

HOPI: According to patient he was alright 2 weeks back then he develops fever with rigors with sweating also associated with nausea, vomiting and anorexia also has constipation as well. He takes paracetamol for fever which works. He says that he had mild headache and lethargic. He had no history of dyssuria, haematuria or uregency

PMH: HTN for 1 yrs Familial renal disease DM Asthma Strokes TB Past surgical history: He had a cadaveric renal transplant in 3/1/2000 Left AVF formed on 2/3/98 He received kidney from 21 yrs old male died of head injury Cold ischaemic time 24 hrs Arrest time 50 mins Re-perfussion time 45 times Drug history: Mycophenolate mofetil 500mg BD Neural 125mg BD Amlodipine 5 mg Allergy history: No allergy history

85

Social history: Lives with his family Coping at home indecently Ex smoker smoked for 4 yrs Rarely drink alcohol Family history: He had a family history of CKD (mother and brother) ADPKD

On examination Well oriented comfortatble co-operative JVP not raised Temperature 38.8 BP: 130/65 mm of Hg R/R: 16 br/min HR: 90 b/min Ankle edema: neck stiffness Respiratory: Chest clear CVS: Normal heart sound Abdomen: Soft non tender No viseromegaly Normal bowel sound

86

Urinary tract: He had got repeatedly UTI for 2 occation although treated with Augmentin for 14 days

Investigation: Urine dipstick Protein - + PH- 5.0 Blood- +++ Leuco-+ Sp.gravity- 1.015 Hb: 12.2 WCC: 15 Platelates- 223 MCV-86.9 Urea 9.4 Cr-148 Impression UTI

Discussion: ADPKD is a hereditary nephropathy characterized by multiple renal cysts, with slow progressive deterioration of renal function. It often terminates in renal failure and accounts for 2–9% of this population. The symptoms of ADPKD do not generally develop until adulthood; 85% of carriers are asymptomatic until the fourth decade of life . In Dalgard's retrospective study, the mean age for diagnosis of ADPKD was 47.2 years, while Singh reported that the mean age at the start of ESRD treatment was 47.3±15.2 years. There is often a period of more than 10 years when patients with polycystic kidneys develop symptoms of end-stage renal disease. Howard et al. reported that an

87

ADPKD donor kidney with cysts size up to 34 mm may take 12.5 years to develop endstage renal disease after transplantation. Based on the age of a 21-year-old cadaver donor there should be at least a 10-year period of kidney function. Over the past 40 years, great improvements have been made in graft survival with some forms of cyclosporin-based immunosuppression. One-year and 3-year graft survivals of 85 and 75% respectively are possible for normal kidney donations. Furthermore, previous reports showed that polycystic kidneys with normal renal function and preserved renal cortical mass could be used for transplantation and disappearance of the cysts may even occur, as reported by Spees . Ettenger et al. compared mortality between renal cadaveric transplant patients and patients on dialysis who were or were not waiting for transplantation. He found that the mortality rate is lower in the renal transplant patients after 2 years follow-up. In consideration of quality of life with a good functioning kidney and the shortage of donors, a polycystic kidney with good renal function may be used as a donor kidney based on the estimated time for the donor kidney to fail and the graft survival period on immunosuppression.

Reference: [22]

88

Ureteric stent( advantages and disadvantages): case report

Background Increasing attention has focused on improving the acceptability and morbidity of antegrade renal and urologic procedures. Separately, a number of refinements of percutaneous nephrolithotomy have addressed the issue of the tube-related morbidity after surgery . For such surgery, a large track (30 F) is created for the percutaneous working sheath and stone removal. After the procedure, the track is exchanged for a 24– 30-F percutaneous tube—a nephrostomy catheter—to ensure satisfactory renal drainage and to tamponade the track. Because of the size and rigidity of the nephrostomy tube, postoperative pain control can be a problem. Some workers have attempted to minimize this tube-related morbidity and advocate tubeless percutaneous nephrolithotomy ; they report that in many cases the protective nephrostomy tube is unnecessary, and the sheath could be removed after the procedure if satisfactory ureteral drainage was assured and bleeding was only minimal. For antegrade ureteral stent insertion, attention has focused on shortening the duration of the procedure. Traditionally this was a two-stage procedure. After relief of ureteral obstruction with a nephrostomy catheter (8–10) and a few days of satisfactory external drainage, an antegrade stent was inserted in a second procedure. Thus, an individual stent insertion episode may last 2–5 days in total. This two-stage procedure or secondary ureteral stent placement is being increasingly superseded by primary antegrade stent placement. In primary antegrade stent placement, primary renal access and stent insertion are performed as a single procedure, and this single procedure has an 80% reported success rate . As originally described , even this procedure required a covering nephrostomy catheter to guard against poor stent function and was removed after 12–24 hours of satisfactory ureteral drainage. From the patient’s perspective, an antegrade ureteral stent procedure was still relatively prolonged—a stent was inserted on day 1 with a nephrostomy tube left in situ, and the covering nephrostomy tube was removed on day 2; the patient either stayed in the hospital overnight or required two outpatient visits to the interventional radiology department. However, if the encouraging experience of tubeless 89

percutaneous nephrolithotomy could be extended to the antegrade ureteral stent procedure, the latter promises to be a true one-stage or tubeless stent service, and the patient would visit the interventional suite only once. The attraction of such a one-stage procedure is obvious, but advantages may be outweighed if they are accompanied by an increased risk to the patient or a higher complication—in particular increased bleeding or septicemia—rate. The purpose of our study was to evaluate the technical success and complications of onestage antegrade ureteral stent placement

Case summary: A 28 years old female student, get admitted in Northern General Hospital with complain of UTI and for removal of ureteric stent. Live kidney related transplant recurrent hydronephrosis of transplanted kidney which nephrotomy done Balloon dialatation of ureter on 13/06/07 and nephrostomy taken out and ureteric stent inserted 13/06/07

Case report: Name: D S DOB: 27/3/79, 28 yrs, female student Presenting complains: Elective admission for removal of ureteric stent

PMH: Live kidney related transplant recurrent hydronephrosis of transplanted kidney which nephrotomy done Balloon dialatation of ureter on 13/06/07 and nephrostomy taken out and ureteric stent inserted 13/06/07 90

Psychriatric problems No cough, shortness of breath, fever, chest pain, vomiting, diarrhea, Had history of transplant kidney for that account hydroureter with impaired kidney function Cr 195 25/5/07 Transplant nephrostomy was performed under LA and it started to drain 5 litres and level of creatinine down to 117 Also admitted for ballon ureteroplasty stent placement on 12/06/07 Had C/0 dysuria 21/6/07 Urine MCS 14/06/07- enterococcus S---trimethoprine, amoxicillin, nitrofurantoin, cefuroxime R---fefalexin, feroroxime Urine MC+S--- mixed growth suggesting contamination Allergy history: NAD

Drug history: Loratadine 10mg Olanzapine 5 mg Tacrolimus 1 mg Nystatin 100,000 U/ml suspension MMF- 250 mg Omeprazole 20 mg Levothyroxine sodium 50 mcg Prednisolone 2.5 mg Seena 7.5 mg Atenolol 25 mg

91

Investigation: Hb: 10.9, K- 4.1, HCO3- 21, PO4- 1.2, Alk PO4- 74, albumin- 34 WCC: 6.8 Urea-7 Cr-115 Urate: 322 On examination: Oriented co-operative but very anxious about her disease and removal of ureteric stent. BP- 116/73 mm of Hg Pulse- 78 b/min Sats- 98% on air Afebrile CVS: JVP not raised Heart sound normal Respiratory: Trachea centrally placed No added sounds GIT: Soft non tender abdomen Bowel sound is audible Stent is still in Enterococcus recently UTI

92

Discussion: Ureteral stent placement is a routine safe procedure for the maintenance of ureteral patency. A stent can be inserted through either the antegrade or retrograde route through a cystoscope. Antegrade insertion is successful in 88%–96% of cases , but the two routes have not been critically compared in an unselected randomized fashion. Although the retrograde route is usually the preferred first option because the potentially serious complications related to renal puncture may be prevented, the final choice often depends on local skills and equipment. However, in some patients, antegrade stent insertion is more likely to succeed, particularly if the ureteral orifices are poorly visualized—for example, in patients with pelvic, bladder, or prostate malignancy. In one study, retrograde stent insertion failed in 14% of all cases, but the failure rate was higher in those with malignant or external obstruction than in those with benign obstruction—27% versus 6%, respectively . In our institution, there is a policy of first attempting antegrade stent placement in all patients with malignant obstruction, whereas patients with benign obstruction, in particular pelviureteric junction obstruction, undergo retrograde stent placement. The major disadvantages of antegrade ureteral stent insertion are that it requires percutaneous renal puncture and traditionally was a two-stage procedure. With improvements in guide wires and stent design, stent placement in obstructed but noninfected ureters can be performed safely as a single procedure and is becoming the preferred option of many interventional radiologists. The clinical value of this so-called primary antegrade ureteral stent procedure (ie, renal access and stent insertion carried out as a single procedure) has been demonstrated in a separate group of patients , and it was successful in 80% of selected cases with a 2.5% major complication rate. However, even this modification required an overnight or approximately a 12-hour hospital stay, as a drainage nephrostomy tube was left in situ, and so patients still visited the radiology department a second time for tube removal. Complication:

93

The major complication was septicemia, and both patients in whom this complication occurred had received gentamicin alone intravenously as a single infusion before the procedure. Some consider gentamicin alone to be inadequate , and we are discussing a new policy regarding antibiotics with our hospital microbiology and infection control department. There were further predisposing factors in both cases—the first patient had undergone bladder surgery recently, and the second patient had undergone unsuccessful retrograde ureteral instrumentation. An indwelling bladder catheter was in place in both cases. Apart from sepsis, another major risk after percutaneous renal intervention is bleeding. No patient in the present study developed major bleeding, and we have not yet seen major bleeding that required embolization after ureteral stent placement in our practice, although one patient who was not part of this study required transfusion. In a standards-of-practice document from the Society of Cardiovascular and Interventional Radiology, a serious hemorrhage risk of 1%–4% is noted, although this risk refers to nephrostomy tube insertion alone . Before the start of this study, bleeding was our particular concern, as a freshly created 8–9-F renal puncture was to be left without a catheter to tamponade the track. However, we were persuaded that tubeless percutaneous urologic procedures were safe because of the encouraging experience after so-called tubeless percutaneous nephrolithotomy. It has long been clear that the large nephrostomy catheter—typically 24 F—left as a drainage catheter after even uncomplicated percutaneous nephrolithotomy results in marked minor postprocedure morbidity, which is related to discomfort and pain caused by the tube. To improve procedural morbidity, interest has focused on elimination of the postoperative nephrostomy tube if the procedure is uncomplicated.

Conclusion: One-stage tubeless antegrade ureteral stent placement is feasible in carefully selected patients and will be successful in more than 80% of all patients, including outpatients. It is now our policy to attempt one-stage stent placement in all suitable cases. Until there is

94

further experience, this technique should be reserved for stable and otherwise fit patients with normal results of hematologic studies. In our study, major complications were all infections. Patients who are clinically suspected of having infection should be excluded, as should those with a predisposition to infection (eg, history of recent retrograde ureteral instrumentation, failed retrograde stent insertion, or genitourinary tract surgery). We continue to be conservative in our approach to clinically unstable patients or those who are suspected of having infected systems. In our view, these patients should still undergo secondary antegrade stent placement after initial nephrostomy tube drainage has resolved the pyonephrosis and the clinical status improves. In any case, careful technique is very important, and all patients should receive preliminary antibiotic therapy. If a patient is suspected of having any periprocedural bleeding or substantial clotting is seen in the collecting system on a postprocedural nephrostogram, then a temporary nephrostomy catheter should be inserted. Even if one-stage stent placement has been successful, cases should be reviewed daily for the next few days either in the ward or by means of a telephone interview if the person is an outpatient

Reference: [23]

95

ATN after Transplantation:report case

Background Kidney transplantation is the best option for all patients with terminal renal failure. Kidney transplantation is not only associated with an improved quality of life in comparison to all other renal replacement therapies, this method also offers a significantly extended lifespan. Therefore, the option for transplantation has to be verified for every patient with renal failure. Graft and patient survival is best when transplantation is carried out just before starting dialysis treatment. Realistically, only living donor transplantation offers the option of sparing the recipient a long waiting period on dialysis. Although transplantation from living donors is superior to cadaveric kidney transplantation, a small risk remains for the donor. Kidney transplantation and the immunosuppressive therapy are associated with an increased risk for certain types of infection, an increased tumour risk and an increased risk for carciovascular complications. To address these problems, specific recommendations for patient surveillance have been provided by different transplantation societies.

Case summary: A 44 years female admitted in Northern General Hospital with complain shortness of breath, ankle swelling, cough with sputum. Patient been well recently then suddenly developed cough with sputum and shortness of breath. She also complained ankle swelling and chest pain some times. Recently she is anuric for 1 week she has no history of MI or angina. Her past medical history was significant for systemic lupus erythematosus (SLE) for 7 years when she presented with fever, rash, polyarthralgia and anasarca and Cadaveric renal transplant had done on 2003. she started hemodialysis on RUE 9/02/99.

96

Case report: Name: G J Sex: female Age: 44 yrs Hospital no: 604391 Present complain: Shortness of breath Ankle swelling Cough with sputum

HOPI Patient been well recently then suddenly developed cough with sputum and shortness of breath. She also complained ankle swelling and chest pain some times Recently she is anuric for 1 week she has no history of MI or angina

PMH Cadaveric renal transplant 2003 SLE- for 7 yrs HTN- 10 yrs Deteriorating renal function for 5 yrs Hemodialysis on RUE 9/02/99 Right elbow AVF on 2/2/99 Transplanted right kidney biopsy 25/6/03 Family history: Nothing significant Social history: 97

Lives with son Works in electronic firm Smokes 10/day cigarettes

Drug history: Venofer 100mg Calcium resorum 15mg Alfacalcidol .5microgram Prednisolone 5 mg Renagel Descloradine 5mg Calcium acetate Lansoprazole 15 mg Quinine sulphate 300mg Fluamcorhzone 100 microgram Clonazepam 500 microgram Aspirin 75mg

On examination BP-121/84 mm of Hg Pulses: 74 b/min R/R: 16 br/min Sats 90% on air Heart sound normal Pedal odema ECG Sinus rhythm Blader no obvious distension Mild sacral odema Drain site healthy

98

Bruit over transplanted kidney Bowel sound normal

Investigation Hb 9.9 WCC: 7.1 Platelates- 349 Urea- 9.4 Cr- 409 HCO3- 31 Ca- 2.18 PO4- 1.49 Had renal transplant on 31/7/07 by Mr Halawa then after she has no pain MAG scan 3 and anuric Input- 3.7 lts Output: nil Investigation after surgery Hb : 14.3, WCC: 7, MCV: 102, K: 5.4, urea: 4.1, Cr- 406

Diagnosis ATN–

Discussion: Previous studies aimed at identifying the causes, risk factors, and outcome of kidney transplant recipients with delayed graft function (DGF) have yielded controversial results. We retrospectively analyzed the causes and risk factors for DGF in 263 cadaveric kidney transplantations from November 1988 to March 1997 in one center. Causes of 99

DGF were assessed by postoperative graft evolution and graft biopsy. Univariate and multivariate analysis were used to investigate the risk factors for DGF induced by acute tubular necrosis (ATN). Seventy-six patients (29%) had DGF, which was caused by ATN in 70 patients (92.1%) and acute rejection (AR) in 6 patients (7.9%). Therefore, we focused on risk factors and consequences for ATN-induced DGF. In monofactorial analysis, ATN was significantly associated with greater weight and presence of an atheromatous disease in both donor and recipient. Other risk factors for ATN were older age of donor, recipient American Society of Anesthesiology (ASA) physical status category IV, cold ischemia time (CIT), and transplantation using the right kidney. The multivariate analysis showed that donor and recipient weight, donor age, transplantation using the right kidney, preservation in Eurocollins solution, ASA score, and CIT were associated with ATN. The incidence of rejection and renal function were not different at 3 months or 1 and 5 years. ATN is the main cause of DGF in kidney transplant recipients. ATN is caused by donor and recipient vascular background, grafting the right kidney, and CIT. ATN does not appear to have an adverse effect on long-term kidney function, hydrocarbon solvents, organic solvents, herbicides, bacterial and viral infections In conclusion, recurrent disease, although rare, can occur at any time after transplantation and should be considered with late graft dysfunction. There is often, but not always, an identifiable precipitating factor. A high index of suspicion is the key for early diagnosis because it has been shown that prompt treatment can salvage the graft.

Reference: [24]

100

Psychotic disorders, after transplant: case report

Background Patients with psychotic disorders have been considered appropriate candidates for transplantation but are expected to have special needs. Nonetheless, the special needs of these patients are unclear and will most likely depend upon the individual circumstances of each patient and their previous psychiatric history. In a 1993 survey of transplant programs, only 33% of heart, 15% of liver, and 6% of renal transplant programs indicated that controlled schizophrenia was an absolute contraindication to transplant..

Case summary: A 21 years old girl admitted in Northern General Hospital with complain of feeling sickness and admitted here for renal biopsy. Patient had history of maternally donated renal transplant on 2/7/03. she was living with parents and also attend college. She had long and complex medical history that she first presented in 2002 with ESRD cause unknown. A renal biopsy showed ESRD without any evidence of glomerulonephritis. There was an intriguing past history of her having been investigated as an infant with dysmorphic feature and apparently abnormal looking kidney of ultrasound. A repeat USS performed by an expert paediatric radiologist was however normal. At initial presentation with end stage of renal disease, Tegan also had an extremely severe dialated cardiomegaly with profound symptomatic hypotension and severe reduced ejection fraction. She under went a living related donor transplant after 18 months on peritoneal dialysis, however unfortunately she had a number of very major and debilitating problems. Early post transplant she developed acute psycosis that was sufficient in severity. She had to be transferred for in patient psychiatric management.

101

Case report: Patient name: T C Age: 21 yrs Sex: female Present complain: Feeling sick today Also have little sickness Admitted here for renal biopsy.

HOPI: Patient had history of maternally donated renal transplant on 2/7/03. she was living with parents and also attend college. She had long and complex medical history that she first presented in 2002 with ESRD cause unknown. A renal biopsy showed ESRD without any evidence of glomerulonephritis. There was an intriguing past history of her having been investigated as an infant with dysmorphic feature and apparently abnormal looking kidney of ultrasound. A repeat USS performed by an expert paediatric radiologist was however normal. At initial presentation with end stage of renal disease, Tegan also had an extremely severe dialated cardiomegaly with profound symptomatic hypotension and severe reduced ejection fraction. She under went a living related donor transplant after 18 months on peritoneal dialysis, however unfortunately she had a number of very major and debilitating problems. Early post transplant she developed acute psycosis that was sufficient in severity. She had to be transferred for in patient psychiatric management.

PMH: ESRF due to unknown cause. 2002 CAPD 2002-03 Dialated cardiomyopathy 2002 Spontaeniously live transplant from mother 2003 Psychosis post transplant

102

Personal history: She is living with parents Social history: She is no smoker and non alcoholic

Drug history: Amlodipine 10 mg OD One alpha 0.25 mcg OD Frusemide 160 mg OD Calcichew 1.2gm TDS NaHCO3 50 mg BD Neorecormon 4000 IU 2/week

On examination Ill looking, BP-121/84 mm of Hg Pulses: 74 b/min R/R: 16 br/min Respiratory: Chest clear

CVS Heart sound normal Abdomen Soft non tender

103

Investigation: Hb 10.8 gm WCC- 5.6, Plts- 168000, HCT- 0.38 Na-138, K-4.9, Urea-38.6, Crt-474, HCO3-18, Ca 2.21, PO4- 1.8 Bone mineral density report: Osteopenia Fracture history nil

Discussion: This patient with acute psychosis was felt to be an appropriate candidate for a renal allograft. She had a good adjustment to dialysis, and despite his chronic psychiatric disorder the medical and psychiatric staff felt that he could comply with the transplantation medications. Overall, the transplant team decided that the benefits of renal transplantation outweighed the potential problems in the case of this high-risk patient. Since patients with chronic psychotic disorders rarely present as candidates for renal transplantation at our institution or elsewhere in the nation, few transplant programs have developed a process to specifically address their complex psychosocial needs. The only other published report regarding transplantation recipients with schizophrenia discusses two patients who underwent elective removal of a transplanted organ. One patient received a cadaver renal transplant, which was removed when the patient refused to take his ant rejection medication or permit laboratory studies. This patient was known to have ongoing alcohol dependence and was felt not to understand the diligence required for ongoing transplant care. The second patient was a man with insulin-dependent diabetes mellitus who received a cadaver pancreas transplant. The patient requested removal of the transplanted pancreas because he felt it compromised his quality of life. The transplant was not removed, and the patient followed up with outpatient psychiatric care. This patient was felt to be compliant with ant rejection medications. The authors

104

report that in both of these cases, family members had pressured the patient to receive the transplant. In my case, there was no family pressure on the patient to accept the transplant. She had been advised that long-term dialysis presented greater risk of long-term medical complications than transplantation. She had consented to undergo transplantation based on these considerations plus his desire not to spend time receiving dialysis. The limited family support was evidently replaced by the considerable support from medical and nursing staff at the medical center clearly; patients must make their own informed decision about a procedure of this magnitude. It is our assertion that patients with schizophrenia benefit from transplantation but may require additional monitoring and treatment planning to address their psychiatric, medical, and psychosocial needs. The fact that this patient developed a bond with the dialysis staff is very promising because it demonstrates his ability to develop a therapeutic relationship. In this case, I believe that her positive adaptation to dialysis bodes well for his participation in a supervised, supportive group therapy program while adhering to his post transplant medical care

Reference: [25]

105

Delayed graft function and acute tubular necrosis (ATN): case report

Background Delayed graft function (DGF) is a common clinical problem occurring after cadaveric renal transplantation. DGF occurs in more than half of the cadaver grafts Acute tubular necrosis (ATN) is one of the main causes of the DGF with an incidence varying from 20% to 50% in the cyclosporine era. Protracted recovery of ATN may continue for several weeks. Persistence of oliguria in ATN is an ominous sign that usually results in allograft loss or chronic renal insufficiency. Here, we describe a case of prolonged oliguric ATN that lasted for more than three months with complete recovery of renal function.

Case summary: A 32 years girl, get admission for biopsy purpose. Today at evening will go for HD and will plane to go home foe a break then come back Sunday for HD. She informed that she has been adviced to drink more water to pass urine, she went under cadaveric transplant. Since starting CAPD in January 1998. she has had two episode of peritonitis although these have both clear up well. It would seem time to put her on the transplant operation. We have mentioned in passing to her family that there is always the possibility of a live related donor but have not looked in to this any further. She under transplant 15/7/07. she had biopsy last Monday 23/07/07 She had past medical history of Two small kidney, Recurrent UTI in children HTN, CAPD, recurrent peritonitis

Case report: Name of the patient: N C Age: 32 Present complain:

106

Today she is feeling fine and well. Feeling bored in hospital

HOPI: Today at evening will go for HD and will plane to go home foe a break then come back Sunday for HD. She informed that she has been adviced to drink more water to pass urine, she went under cadaveric transplant. Since starting CAPD in January 1998. she has had two episode of peritonitis although these have both clear up well. It would seem time to put her on the transplant operation. We have mentioned in passing to her family that there is always the possibility of a live related donor but have not looked in to this any further. She under transplant 15/7/07. she had biopsy last Monday 23/07/07

PMH Two small kidney Recurrent UTI in children HTN CAPD, recurrent peritonitis Personal history She is living with parents Social history She is smoker and drink regularly Family history NAD

107

On examination BP- 135/94 mm of Hg Pulse- 90 Left Sacral odema CVS Heart sound normal Respiratory Chest clear Abdomen Soft non tender

Investigation USS kidney shows 24/7/07 normal

Diagnosis ATN Tx AVF

Discussion: Renal failure persisting after renal transplantation is called delayed graft function (DGF). The definition of DGF is varied in different studies, but generally refers to anuria or requirement for dialysis after the first week post transplantation. Less than 5 percent of the kidneys with DGF never function (primary nonfunctioning). The frequency of DGF may be as low as 10% and as high as 70% in some series.

108

The major causes of DGF are acute tubular necrosis (ATN), hyperacute rejection, accelerated rejection superimposed on ischemic ATN, urinary tract obstruction or, very rarely, atheroembolic or thrombosis of the renal artery or vein. ATN is the most common cause of DGF with an incidence varying from 20% to 50% in the cyclosporine era. Causes that have been implicated in the etiology of ATN include renal ischemia due to hypoperfusion in the donor, prolonged warm and cold ischemia times, harvesting conditions, surgical procedures and cyclosporine given immediately following transplantation. Cyclosporine (especially at doses above 10 mg/kg per day) and prior sensitization in the retransplanted patients have been shown to increase the incidence and duration of ATN. Some investigators have reported an increased risk of delayed graft function following transplantation in the cyclosporine treated patients. However, others have failed to document any difference in the rate of DGF with the conventional immunosuppressive drugs. ATN tends to become prolonged when cyclosporine is used in these settings. Administration of calcium channel blockers can prevent the acute renal vasoconstriction induced by cyclosporine., The long-term reno-protective effects of these agents have yet to be proven. Chung et al treated 13 patients with DGF and ATN with a low dose cyclosporine and found no significant decrease in the one-year survival rates. However, he found that poor functional recovery of ATN or non-ATN condition could predict poor one-year allografts survival rate. The most dangerous potential risk of the low dose cyclosporine used in the patients with DGF was the increased incidence of rejection that results in poor graft survival. On the other hand, the episodes of infection were significantly lower. In our patient, the prolonged ischemia time resulted in primary nonfunctioning allograft. Cyclosporine was not included in the induction regimen in an attempt to decrease the duration of the DGF. The initial ischemic tubular injury was worsened by the contrast media-induced nephrotoxicity. Angiogram had to be performed to rule out vascular occlusion as was evident on Doppler ultrasound. The fulminant chest infection with

109

pulmonary failure obligated the use of aminoglycosides, adding further to the tubular injury. Furthermore, the biopsy of renal allograft was resorted to on three different occasions to evaluate the delayed function and to rule out acute rejection. These repeated and continued insults to the allograft might have contributed to the lengthy ATN that required the prolonged dialytic support.

Reference: [24]

110

Post-transplant IgA nephropathy: case report

Background Currently, post-transplant IgA nephropathy (IgAN) is considered to include recurrence of IgAN, de novo IgAN, or transmission of IgA deposit from donor kidney . Yamaguchi et al. reported that the prevalence of post-transplant IgAN was 34% in renal biopsies of allografts surviving more than 5 years. Berger et al. were the first to describe recurrence of IgA deposit, and subsequent studies showed 50–60% prevalence of recurrent IgAN. Graft loss due to recurrent renal diseases has been reported to be a rare event, estimated as less than 5% However, it has recently been shown that deteriorating renal function and substantial graft loss were observed in 55% of renal allograft recipients with recurrent IgAN at long-term follow-up It appears that normal blood pressure is a good marker of graft survival and that an effective antihypertensive treatment reduces the progression of graft damage. Protection of allografts from long-term functional deterioration is one of the major goals of treatment of post-transplant patients. In primary IgAN, hypertension and proteinuria (more than 1 g/day) are considered to indicate a poor prognosis. Numerous studies have shown that angiotensin-converting enzyme (ACE) inhibitors interfere with progressive deterioration of many chronic renal diseases, including IgAN. Only one report so far has shown the antiproteinuric efficacy of ACE inhibitors in patients with post-transplant glomerulonephritis. The beneficial effect of ACE inhibitors still remains obscure for post-transplant IgAN, and to date there are a few in-depth studies in renal transplantation that have analysed the relationship between histological lesions and response to ACE inhibitor treatment.

111

Case summary: A 58 years british male, admitted in Northern General Hospital with complain of Mild diarrhea for 5 days 8-10 times per day. Patient admitted for a renal transplant which he had on the 18/05/07. Post operation cause was relatively uneventful. His pre operative Cr was 870. He had a DVT following air journey and was found to have impaired renal function. He has had a biopsy in Nottingham with a possible histological diagnosis of IgA nephropathy. He was admitted with uremic pericarditis, dialysed briefly and was put on CAPD. He did not get adequate dialysis through it and he has been switched back HD on renal RUE through a tunnel line neck since may 2003 He was clinically fairly fit and well with no history suggestive of IHD or peripheral heart disease. He also informed that time shortness of breath at about 100-150 yars He had past medical history of ESRD- hospital haemodialysis IgA nepthropathy, Controlled HTN, renal transplantation

Case report: Name of the patient: C C Sex: male Age: 58 Present complain: Mild diarrhea for 5 days 8-10 times per day Feeling well

HOPI Patient admitted for a renal transplant which he had on the 18/05/07. post operation cause was relatively uneventful. His pre operative Cr was 870. He had a DVT following air journey and was found to have impaired renal function. He has had a biopsy in Nottingham with a possible histological diagnosis of IgA

112

nephropathy. He was admitted with uremic pericarditis, dialysed briefly and was put on CAPD. He did not get adequate dialysis through it and he has been switched back HD on renal RUE through a tunnel line neck since may 2003 He was clinically fairly fit and well with no history suggestive of IHD or peripheral heart disease. He also informed that time shortness of breath at about 100-150 yars

PMH ESRD- hospital haemodialysis IgA nepthropathy Controlled HTN Transplantation Personal history: Not smoking since 2003 Ex security guard Allergy history No allergy history Family history NAD Social history He was regular alcoholic but now he informed occasionally drink

On examination: BP- 135/94 mm of Hg Pulse- 90 Left Sacral odema

113

CVS: Heart sound normal Respiratory Chest clear Abdomen Soft non tender

Diagnosis IgA nephropathy

Discussion: In the present study we have studied the histological characteristics of post-transplant IgAN and primary IgAN. As far as we are aware, this is the first systematic analysis of histopathology of post-transplant IgAN. The main findings are as follows: (i) acute lesions such as mesangial proliferation, crescent formation were rarely found in posttransplant IgAN; (ii) chronic lesions such as glomerular obsolescence and interstitial fibrosis were observed irrespective of the severity of the glomerular changes; (iii) in posttransplant IgAN, glomerular hypertrophy was observed at any grade of glomerular obsolescence. These histopathological characteristics may result from the pathophysiological conditions of the allograft kidneys. In renal allografts, glomerular hypertrophy was reported to be a risk factor of focal glomerulosclerosis and a predictor of risk of late allograft dysfunction .Focal and segmental sclerosis is also thought to result from glomerular hypertension/hyperfiltration and is also considered to be related to proteinuria. In addition we found that segmental sclerosis was more frequently found in the patients with heavier proteinuria (more than 1.0 g/day).

114

It has become evident that several non-immunological factors such as systemic hypertension, glomerular hypertension, and heavy proteinuria may affect long-term graft outcome .In patients with a renal allograft, a state of hyperfiltration, one of the main nonimmunological factors, can occur secondarily to a reduction in the functioning renal mass by various circumstances (e.g. repeated acute rejection episodes, CsA nephrotoxicity, small renal graft size, or post-transplant glomerulonephritis). Thus, hyperfiltration is considered to play an important role in the progression of renal insufficiency in the late post-transplant period.

Conclusion In conclusion, in post-transplant IgAN, histopathological lesions indicative of acute inflammatory insults were suppressed; but glomerular hypertrophy, which relates to haemodynamic burden such as hyperfiltration, was prominent. These histological features support the use of ACE inhibitors in the treatment of hypertension/hyperfiltration in posttransplant IgAN. Preliminary study of ACE inhibitor treatment in 10 patients showed favourable effects. Future long-term follow-up studies are required to establish the effectiveness of ACE inhibitors in the treatment of post-transplant IgAN

Reference: [11]

115

1.

Carolyn W. Van Dyke, Thomas J. Masaryk, and P. Lavertu, Multiple Myeloma Involving the Thyroid Cartilage. AJNR, 1995: p. 570-572.

2.

Władysław Sułowicz, et al., Acute renal failure in patients with rhabdomyolysis. Med Sci Monit,, 2002.

3.

Nobuo Tsuboi, et al., Acute Renal Failure after Binge Drinking of Alcohol and nonsteroidal antiinflammatory drug ingestion. 1997. 36: p. 102-106.

4.

Singh S, et al., Acute alcoholic myopathy, rhabdomyolysis and acute renal failure : a case report. neurology india, 2000 48: p. 85-5.

5.

Ravat SH, et al., Malignant Hypertension due to Reflux Nephropathy in an Aldolescent ( A Case Report). J Postgrad Med, 1991. 37(1): p. 173-176.

6.

Besarab1, A., Resolving the paradigm crisis in intravenous iron and

erythropoietin management. International Society of Nephrology, 2006. 69: p. S13–S18. 7.

Muirhead, N., Individualizing anaemia treatment: a discussion of case histories. Nephrology Dialysis Transplantation, 2005. 20: p. 37-43.

8.

F Turgut1, M.K., B Isik2 and A Akcay, Risk factors affecting the

incidence of chronic kidney disease Kidney International,, 2007. 71: p. 15166. 9.

WILLIAM F. KEANE, B.M.B., 2 DICK DE ZEEUW,3 JEAN-PIERRE GRUNFELD,4, et al., The risk of developing end-stage renal disease in patients with

type 2 diabetes and nephropathy: The RENAAL Study. Kidney International, 2003. 63: p. 1499–1507. 10.

Yamagata, K.,

Response to ‘Risk factors affecting the incidence of chronic kidney disease’. Kidney International,, 2007. 71: p. 1076-1078. 116

11.

ALEXOPOULOS, E., Treatment of primary IgA nephropathy. Kidney International,, 2004. 65: p. 341-355.

12.

A Chang1, et al., Membranous and crescentic glomerulonephritis

in a patient with anti-nuclear and anti-neutrophil cytoplasmic antibodies. International Society of Nephrology, 2007. 71: p. 360–365. 13.

T Thiruventhiran, et al., Acute renal failure following multiple wasp stings. Nephrology Dialysis Transplantation, 1999. Vol 14(Issue 1 214-217).

14.

BESSIE A. YOUNG, M., MPH, P. CHARLES MAYNARD, and M. EDWARD J. BOYKO, MPH, Racial Differences in Diabetic

Nephropathy, Cardiovascular Disease, and Mortality in a National Population of Veterans. P a t h o p h y s i o l o g y / C o m p l i c a t i o n s, 2003. 26: p. 2392–2399,. 15.

J.Falk, R., ANCA-associate renal disease. Kidney International,, 1990. 38: p. 9981010.

16.

Matsumoto, Y.T.a.A., Reversible Posterior Leukoencephalopathy Syndrome

in p-ANCA-associated Vasculitis. The Japanese Society of Internal Medicine, 2006. 334: p. 494-500. 17.

PE Stevens1, D.O.D., S de Lusignan3, J Van Vlymen4, B Klebe5, R Middleton2, N Hague6, and J.N.a.C. Farmer, Chronic kidney disease management in the United

Kingdom: NEOERICA project results. International Society of Nephrology, 2007. 72: p. 92-99. 18.

G, L.A.E.K.T.Y.L.A.C.J.R.J.Z.D.H.T.L.N.E., Definition and classification of chronic kidney disease: A position statement from Kidney Disease: Improving Global Outcomes (KDIGO). Kidney International,, 2005. 67(6):: p. 2089-100.

19.

AS, J.T.S.P.S.C.L.M.M.G.d.J.P.d.Z.D.S.S.T.R.L., Progression of chronic kidney disease: the role of blood pressure control, proteinuria, and angiotensinconverting enzyme inhibition: a patient-level meta-analysis. Ann Intern Med 2003. 139(4):: p. 244-52.

117

20.

Roe, A.G.B.a.S., Wegener's granulomatosis presenting as a pleural effusion. BMJ, 2003;. 327: p. 95-96.

21.

Nelson Mazzuchi, F.G.-M., Enriqueta Carbonell, comparison of survival for haemodialysis patient vs renal transplant recipient treated in Uruguay

Nephrology Dialysis Transplantation, 1999. 14: p. 2849-2854. 22.

yan-Shen Shan, P.-C.L., Edgar D. Sy, Chung-Jye Hung and and Y.-j. Lin, polycystic kidney patient as a cadaveric donor: is it appropriate? Nephrology Dialysis Transplantation, 2001. 16: p. 410-411.

23.

Uday Patel, M., ChB,, F. MRCP, and M. M. Ziyad Abubacker,

Ureteral Stent Placement without Postprocedural Nephrostomy Tube: Experience in 41 Patients1. Kidney, interventional procedures,, 2004. 230: p. 435-442. 24.

Huraib Sameer, A.K.W., Al Ghamdi G, Iqbal A, Post Transplant Acute Tubular Necrosis - How Long you can Wait?: A Case Report. Saudi Journal Kidney Disease Transplantation, 2002. 13 |(1): p. 50-54.

25.

LOIS E. KRAHN, M.D., M.D. GREGORY SANTOSCOY, and M.D. JON A. VAN LOON, A Schizophrenic Patient’s Attempt to Resume

Dialysis Following Renal Transplantation. The Academy of Psychosomatic Medicine. , 1998: p. 470-473.

118

Related Documents

Clinical Dissertation
April 2020 6
Dissertation
May 2020 36
Dissertation
August 2019 93
Dissertation
May 2020 35
Dissertation
November 2019 45
Dissertation
October 2019 39

More Documents from ""

Clinical Dissertation
April 2020 6
Cover Heg.docx
December 2019 28
Pengesah Heg.docx
December 2019 36
Oke Fix.docx
December 2019 43